Sunteți pe pagina 1din 151

INAO Question Papers

(2008-2014)

Compiled By
Science Olympiad Blog

A-PDF Watermark DEMO: Purchase from www.A-PDF.com to remove the watermark

Indian National Astronomy Olympiad 2008


Junior Category

Roll Number:

Roll Number

Model Solutions
Date: 2nd February 2008
Maximum Marks: 100

INAO 2008
Duration: Three Hours
Please Note:

The examination consists of three parts. This question booklet containing parts A and B
should be returned to invigilators at the end of 2.5 hours. At that time, second question
booklet containing part C of the paper will be given to you. You will get 30 minutes to
solve part C.
m

oom
In part A and part C, there are 20 multiple choice
.c.c questions each. For each question,
only one of the four alternatives is correct. Mark
the correct answer on the answer sheet
a
amarks
l +3
l
provided separately. Each correct answer adds
to your score. In part A, every
ee
wrong answer carries penalty of -1 marks. There is no negative marking in part C.
m
m
d
d
In part B, there are 4 analytical questions
aa of 10 marks each. The answer to each question
must be written in the blank space
oprovided below each question.
lo
l
For the rough work, use the n
page(s)
n marked as rough sheet.
ww
oo
Only non-programmable
calculators are allowed.
d
d
.
.
Return BOTH the question
paper booklets and the answer sheet to the invigilator. DO
ww
NOT TAKE THIS
BOOKLET BACK WITH YOU.
ww
ww
Please fill
in all the data below correctly. The contact details provided here
would be used for all further correspondence.

Full Name (BLOCK letters) Ms. / Mr.:

Male / Female

Date of Birth (dd/mm/yyyy):

Name of the school / junior college:

Class: VIII / IX / X / XI

Board: ICSE / CBSE / State Board / Other

Full Residential address (include city and PIN code):

Telephone (with area code):


Email address:

for more papers please visit www.downloadmela.com

Instructions for using the answersheet:


Write the name at the top of the answer sheet.
On the left side there is space provided for roll number. Write your INAO roll number in
the squares with exactly one digit per square.
Below each of the digits of roll number, mark corresponding digit by a cross mark (X). i.e.
if your roll number is 40001, then you will put X on 4, 0, 0, 0 and 1 in the corresponding
columns.
Below the roll number, you should mark your preference for Either Astronomy camp or
m
mcorresponding box. Mark only
Junior Science Camp by putting a cross mark (X) in
the
oo
st
one box indicating your 1 preference. You c
will
be
. .c automatically considered for the
st
second preference if the 1 choice is not available.
a

l la
e
Use only black or blue pen to put X marks e
on the answersheet.
m
ink or pencil.
ddm
aa
o
l o
Useful
nnl Physical Constants
w
Mass of Earth
ME
oow
Radius of Earth
RE
dd
.
.
Mass of Sun
M
w
w
Radius of Sun
R
w
Speed of Light w
c
w
Astronomical Unit
1 A. U.
w
Gravitational Constant
Gravitational Acceleration
Speed of Sound (at room temperature in air)

Do not use any other

G
g
cs

5.97 1024 kg
6.4 106 m
1.99 1030 kg
7 108 m
3 108 m/s
1.5 1011 m
6.67 1011 m3 /(Kg s2 )
9.8 m/s2
340 m/s

Space for Rough Work

for more papers please visit www.downloadmela.com

INAO Jr 2008

Part A: Multiple Choice Questions


1. The unit on a graph paper is changed in scale from 1 cm to 1 inch. What will be the
change in area of the unit cell?
(a) 84%

(b) 254%

(c) 545%

(d) 645%

2. Of the Galilean Moons, which is the farthest from Jupiter?


(a) Io

(b) Europa

(c) Ganymede

(d) Callisto

3. When you stand on the ground, what is the distance of the horizon from you?

m
m
o
.cc
A regular barometer is thrown from the topaof .a building.If the barometer is freely
l a
falling, what will be the height of the mercury
eel column?
(a) 100 cm (b) 76 cm (c) 50 cmm
(d) 0 cm
ddm
aathroughout the year, but happen only in certain
P. Eclipses are not distributed evenly
o
l lo
months of a given year.
n
Q. Orbit of the Moon (aroundnthe Earth) makes an angle of roughly 5 degrees to the
ww
orbit of Earth (around o
Sun).
o
Which of the following
options
is correct?
d
.d
.
(a) Statement
wwP is correct but Q is incorrect.
ww P is incorrect but Q is correct.
(b) Statement
ww the statements are correct and Q is the correct reason of
(c) Both

(a) 500 km
4.

5.

(b) 5 km

(c) 15 km

(d) 50 kmo

P.
(d) Both the statements are correct and Q is not the reason of P.
6. When the ball at the end of the string swings to its lowest point, the string is cut by
a sharp knife as shown. Assuming no air resistance, what will be the path of the ball?
Simple Pendulum

String

A
B
Heavy Ball
C
D

for more papers please visit www.downloadmela.com

INAO Jr 2008

(a) A

(b) B

(c) C

(d) D

The answer is (c).


7. If we ever make contact with aliens, which of our fundamental units is likely to match
theirs? (In other words, which of these units is universally fundamental?)
(a) Kelvin

(b) Light year

(c) a.m.u. (Atomic Mass Unit)

(d) None of these

8. If the person beats drum on the Earth and an astronaut beats an identical drum in
space, what will be the differences in the effects?
(a) There will be no vibration in the drum in space.
(b) There will be vibration in space but no sound.

m
(c) The drum on Earth will vibrate for a longer
time than the one in space.
m
o

o
(d) There will be no difference in terms of
c vibrations or sound.
.cthe

9.

10.

aa.
l
Which of the following Venn diagrams would
eelbe BEST suited to represent the three
categories of animals?
m
- Animals that give us MEAT ddm
a
- Animals that give us EGGSo a
l o
- Animals that give us MILK
nnl
w
oow
.d
.d
ww
ww
(b)
(c)
(d)
The answer is (a).
(a)
ww
P: Gravitational
force exerted by Saturn on a human being is approximately same as
that exerted by another human being standing a few cm away.
Q. Saturn has very low density.
(Additional data: Mass of Saturn = 5 1026 kg, Distance of Saturn = 1.4 109 km)
(a) Statement P is correct but Q is incorrect.
(b) Statement P is incorrect but Q is correct.
(c) Both the statements are correct and Q is the correct reason of P.
(d) Both the statements are correct and Q is not the reason of P.

11. For the Earth, if the perihelion were 147 million km, approximately what will be the
aphelion for the Earth?
Aphelion: Point farthest from the Sun in the orbit of a body about the Sun.
Perihelion: Point nearest from the Sun in the orbit of a body about the Sun.
(a) About 2 times the Perihelion, 300 million km
(b) About 3 times the Perihelion, 450 million km
(c) Slightly more than the perihelion, about 155 million km
(d) Exactly the same as the perihelion, 147 million km

for more papers please visit www.downloadmela.com

INAO Jr 2008

12. A star is seen rising from Kolkata (23.5 N 92 E) at 7:00 pm IST, at what time IST
will it be seen to rise from Mumbai (19 N 72 E)?
(a) 5:40 pm

(b) 7:00 pm

(c) 7:20 pm

(d) 8:20 pm

13. To separate various components of air, the principle that different gases have different
. . . . . . . . . can be used.
(a) Boiling Points

(b) Density

(c) Color

(d) Molecular weight

14. How many zeros will be as the ending digits of 120! ? (120! = 1 2 ..... 119 120)
(a) 25

(b) 26

(c) 27

(d) 28

Note: In the original paper, the choices given had typographical errors. Thus, the
correct answer 28 was not listed amogst the options. m
As a result the said question was
removed from evaluation.
oom

.cc

.
15. A nutty professor discovers a way to shrink objects
lala in size using lasers and mistakenly
shrinks his three teenage kids by a factor e
of 100. These kids then stray away into the
e
garden where they see their pet dog. How
many of the kids can climb onto the dog to
m
m
get a ride home if the dog can bear d
ad
weight of 20 kg?
(a) None
16.

a
(c) Two
loo(d) All

(b) One

nnl
Which of the following is true?
ww
oo
(a) cos 80 = sin
10
dd
. cos 240
(b) cos 120 =.
w
w

(c) sin 135


ww= sin 270
(d) sin
330 = sin 210
ww

The answer is (d).

17. A battery is connected by wires to a bulb as shown below and the bulb glows.
Through which points does the charge flow?

1
Battery
4

2
Lighted
Bulb
3

(a) 1-2-3-4-1. Charge flows through the battery also.


(b) 1-2-3-4. Charge flows through the wires and bulb only.
(c) 2-3.Charge flows only through the bulb.
(d) There is no flow of the charge in the circuit.
18. Every object exerts gravitational force on every other object - The force exerted by an
object is higher if its mass is higher. Consider 2 magnets - a bigger magnet P and a
smaller one Q. Which of the following will be true?

for more papers please visit www.downloadmela.com

INAO Jr 2008

(a) Magnet P will exert a greater magnetic force than Q.


(b) The magnetic forces exerted by P and Q will be the same.
(c) Magnet Q will exert a greater magnetic force than P.
(d) We cannot tell from the sizes, as gravity and magnetism are unrelated.
19. What is the value of F in the following equation if A, B, C, D, E and F are non-zero
numbers?
ABCDEF 6 = DEFABC
(a) 1

(b) 3

(c) 5

(d) 7

20. On a cold winter day, if I stand on the edge of a carpet with one foot on the carpet and
one on the smooth granite floor surface, which foot is
mlikely to feel colder and why?

oo heat away from the foot


(a) The foot on the granite as it willcabsorb
. c
more quickly.
a .
l a
e

(b) The foot on the carpet as it will absorb


e l heat away from the foot more quickly.

m
(c) The foot on the granite because
mthe granite is at a lower temperature.
d

aad the carpet is at a lower temperature.


(d) The foot on the carpet because
o
lo
nnl
w
oow
.d
.d
ww
ww
ww

for more papers please visit www.downloadmela.com

INAO Jr 2008

Part B: Analytical Questions


21. A year in Solar calendar consists of 365 days and the same in Lunar calendar consists
of 354 days. The additional days in Solar calendar are kept as balance every year.
Whenever the number of balance days exceeds 30, an additional month of 30 days is
added to the lunar year to offset the difference. The cycle goes on. Anwesh, whose
birthday falls on 1st January, noticed that in the year 2008, his birthday coincided
with the start of the lunar year. In which earliest future year, his birthday will again
coincide with the start of the lunar year? (Ignore leap days.)
Solution: Every year the Solar year lags by 11 days.
Intercalary days are compensated by a month whenever they exceed 30 days.
m
Thus, one has to finde L.C.M. of 11 and 30.
oom
L.C.M. is 330. i.e. after 330 intercalary days.c
are introduced, both calenders will
.c
a
match.
l a
eel
i.e. they will match after 30 years.
Thus, his birthday in 2038 will again
mmark start of the lunar calander.

22.

ddm
aa
o
Note: Brute force method should
l lonot be given more than 7 marks.
nn
w
oow in series, the current in the circuit is 2A. When R1
When R1 and R2 are connected
and R2 are connected
in parallel, the current in the circuit is 4A. Find the values of
.d
.d
R1 and R2 .
ww
ww
w
Solution:w
For a given V,
V

= IA (R1 + R2 )
R1 R2
= IB
(R1 + R2 )
R1 R2
2(R1 + R2 ) = 4
(R1 + R2 )
2
(R1 + R2 ) = 2R1 R2
R12 + R22 = 0
The said conditions cannot be true.
Note: Problem is solvable if one assumes non-zero internal resistance for the battery. Those who reach till previous step, get 9 out of 10. Last point is reserved for
those who take internal resistance into account.

for more papers please visit www.downloadmela.com

INAO Jr 2008

23. You are given two lenses of focal lengths f1 and f2 respectively.
(a) Is it possible to arrange them in such a way that both incident beam and
emergent beam of light will be parallel beams for the following cases?
1. One concave and one convex lens
2. Both convex lenses
Draw the ray diagrams.
(b) Are the incident and emergent beams parallel to each other?
(c) By observing the ray diagrams, state the condition on the distance d between the two lenses in terms of f1 and f2 .

m
oom
1 cc
d
1
1
+
. .
=
f
f1 fl2aaf1 f2
l
d is the distance between the two lenses.ee
m
If the incident beam as well as the emerging
beam are parallel beams, then f1 = 0
m
dfd2
which gives the condition, d = f1a+
a
If one lens is concave and other
convex, it will become, d = f1 f2
loo
Thus, it is only possible if n
focall length of the convex lens is more than that of the
n
concave lens.
ww
oo d is always positive. Hence it is always possible.
If both lenses are convex,
d
. .d
Note: The ray w
diagrams
should be such that above relations could be inferred by
w
w
measuring respective
lengths on the ray diagrams.
w
ww
Solution:

The two beams will be parallel to each other if both lenses are parallel to each
other. Knowledge of the first equation is not expected. Students should inferr the
d = f1 + f2 relation by purely observing ray diagrams.

for more papers please visit www.downloadmela.com

INAO Jr 2008

24. If Aniket starts drawing a straight line with a brand new typical ball-point pen, how
long line can be drawn before he finishes his refill? Explicitly state all the assumptions
you make.
Solution: A typical ball pen refill has length of 12 cm.
(Acceptable 10-15 cm)
Typical refill diameter is 1 mm.
(Acceptable 0.5 - 2 mm)
Hence Total volume of ink is r 2 h = (0.05)2 12 cc
V 0.1 cc
Typical thickness of writing is of the size of finite number of molecules.
Size of one ink molecule can be taken to be 0.5 - 1nm.
m
m
Hence the thickness would be roughly 10nm. oo
.c.c
(Acceptable 1-100 nm)
a
Typical width is half of refill diameter. l la
ee
Thus, length,
2m
r h
0.05 12
V
= d m
=
cm
l=
t d a2td
r
2 10 107
a
l 9.5 km.
loo

nnl
ww
Note: Answer is not o
important
for this order of magnitude
o
Approach to the problem
should be judged for marks.
.d
.d
ww
ww
ww

estimation question.

for more papers please visit www.downloadmela.com

INAO Jr 2008

Part C: Multiple Choice Questions

Roll Number: Roll

Number

25. Resting membrane potential difference is a negative value which means:


(a) No charge outside and negative charge inside of the membrane
(b) Positive charge outside and more negative charge inside of the
membrane
(c) Zero charge inside and negative charge outside of the membrane
(d) Positive charge inside and more negative charge outside of the membrane
26. Nerve impulse is...
(a) Flow of electrons across the axon

m
o
cc
(c) Flow of neuro-transmitters across the. axon
.
a
a
l
(d) Change in pressure across the axon
eel
m
Bitter substances can be tasted in minute
ddmamounts but larger amounts are needed to
taste sweet substances. Based onathis
a observation which of the following reasons is
o
more appropriate?
l lo
n
n are more sensitive than the sweet receptors
(a) The bitter receptors
ww
oo are less sensitive than the sweet receptors
(b) The bitter receptors
d
(c) The bitter. substances
dissolve more easily than the sweet substances
.d
ww
(d) There are more bitter receptors in mouth than the sweet receptors
ww
Gustatoryw
cells
w are stimulated by
(b) Change in ionic constitution acrossothem
membrane

27.

28.

(a) Dissolved chemicals


(b) Pressure
(c) Temperature
(d) Texture
29. The cornea of one person can be transplanted from one person to another with little
or no possibility of rejection as it is beyond the reach of immune system because...
(a) The cornea has no blood vessels
(b) The cornea is a dead tissue
(c) The cornea has no nerve endings
(d) The cornea kills the cells of immune system
30. Find out the mismatched pair in the following combinations.
(a) Nearsightedness longer than normal eyeball
(b) Farsightedness shorter than normal eyeball
(c) Myopia loosened-up extrinsic eye muscles
(d) Astigmatism cylindrical lens

for more papers please visit www.downloadmela.com

INAO Jr 2008

31. Which sequence of events occur when a person looks at a star at night
(a) pupils constrict suspensory ligaments relax lenses become less convex
(b) pupils dilate suspensory ligaments become taut lenses become
less convex
(c) pupils dilate suspensory ligaments become taut lenses become more
convex
(d) pupils constrict suspensory ligaments relax lenses become more convex
32. Which nervous system conducts impulses from CNS to voluntary muscles?
(a) Motor division of PNS

m
oom
.c.c
(d) Parasympathetic division
a
l a
eel
The rate of a simple chemical reaction normally decreases as the reaction approaches
m
completion. This is because
ddm
aa
(a) The reactant molecules
individually
become less active
o
l lo
(b) With the progress n
of the reaction, the temperature goes down and hence the
n
reaction slows down
ww
oinhibit
o the reaction
(c) The products
d
. .d
(d) The concentration
of the reactants decreases
ww
wwis generally used for the ore dressing of
Froth flotation
ww
(a) Oxide ores
(b) Carbonate ores (c) Phosphate ores (d) Sulfide ores
(b) Sensory division of PNS
(c) Sympathetic division

33.

34.

35. The atomic property which is not periodic is


(a) Atomic radius
(b) Mass number
(c) Electronegativity
(d) Ionization energy
36. The largest number of molecules is present in 1 g of
(a) CO2

(b) H2 O

(c) C2 H5 OH

(d) N2 O5

The answer is (c).


37. An isotope of the parent element is produced with the emission of
(a) one - and one - particle
(b) one - and two - particles
(c) two - and one - particles
(d) two - and two - particles

for more papers please visit www.downloadmela.com

INAO Jr 2008

38. A compound was found to contain nitrogen and oxygen in the ratio nitrogen 28 g and
oxygen 80 g. The formula of the compound is
(a) NO

(b) N2 O3

(c) N2 O5

(d) N2 O4

The answer is (c).


39. Acetic acid is a weak electrolyte because
(a) Its molecular mass is high
(b) It is a covalent compound
(c) It is highly unstable
(d) Its ionization is very small

m
m for 100 seconds, deposits
40. A certain current when passed through a CuSO4osolution
o
0.3175 g of copper. The current passed (in A) is
.cc
(a) 4.83

41.

(b) 9.65

(c) 0.965

(d) 0.483laa

eel
For the redox reaction, the correct coefficients
of the reactants for the balanced reacm
ddm
tion are
aa +
2
2+
MnO 4 + C2 O
+ CO2 + H2 O
4 + H Mn
o
l lo
nn
(a) 2
5 16
w
(b) 16 5 2
oow
(c) 5 16 2.dd
.
(d) 2 16w
5
w
ww paper, there were typographical errors in the chemical equation.
Note: In the original
w was removed from evaluation.
Thus, the w
question

42. The pH of 0.1M CH3 COOH (dissociation constant of acetic acid is 1.80 105 at 25
C) will be
(a) 1.0

(b) 2.9

(c) 1.8

(d) 0.2

43. Which kind of number pyramid will fit for the following example?
Grass Deer Flea Leptomonas (parasite of flea).
(a)
(b)
(c)
(d)
The answer is (d).

for more papers please visit www.downloadmela.com

Download ebooks at http://www.downloadmela.com/ebooks/ebookdisplay.html


Download Video tutorials at http://www.downloadmela.com/videotutorials/videotutorials.html
Download placement papers,interview questions,certification dumps and many more at
http://www.downloadmela.com/companypapers/itpapers.html m

oom
Download Previous year papers at
.c.c
a
http://www.downloadmela.com/previouspapers/previouspapers.html
l a
eel
m
ddm
Download Magazines at
aa
http://www.downloadmela.com/magazines/magazines.html
o
l o
nnl
w
oow
.d
.d
ww
ww
ww

for more papers please visit www.downloadmela.com

A-PDF Watermark DEMO: Purchase from www.A-PDF.com to remove the watermark

Indian National Astronomy Olympiad 2008


Senior Category

Roll Number:

Roll Number

Model Solutions
Date: 2nd February 2008
Maximum Marks: 100

INAO 2008
Duration: Two and half Hours
Please Note:

The question paper consists of two parts. In part A, there are 20 multiple choice
questions with 3 marks for each correct answer and -1 mark for each wrong
answer. In each question, only one of the four alternatives is correct. Mark the
m
correct answer on the answer sheet provided separately.
oom Mark a cross (X) in the
corresponding box on the answer sheet.
.cc

aa.
l
In part B, there are 4 analytical questions
eelof 10 marks each. The answer to each
question must be written in the blank
space provided below each question.
m
m
d
For the rough work, use the page(s)
marked as rough sheet.
aad
o
l o are allowed.
Only non-programmable calculators
nnl
ww paper booklet and the answersheet back to the invigReturn the ENTIRE o
question
oTHIS BOOKLET BACK WITH YOU.
ilator. DO NOT TAKE
d
.d
.
w
w
Please fill in all the data below correctly. The contact details provided here
wwwould be used for all further correspondence.
ww

Full Name (BLOCK letters) Ms. / Mr.:

Male / Female

Date of Birth (dd/mm/yyyy):

Name of the school / junior college:

Class: IX / X / XI / XII

Board: ICSE / CBSE / State Board / Other

Full Residential address (include city and PIN code):

Telephone (with area code):


Email address:

for more papers please visit www.downloadmela.com

Instructions for using the answersheet:


Write the name at the top of the answer sheet.
On the left side there is space provided for roll number. Write your INAO roll number in
the squares with exactly one digit per square.
Below each of the digits of roll number, mark corresponding digit by a cross mark (X). i.e.
if your roll number is 40001, then you will put X on 4, 0, 0, 0 and 1 in the corresponding
columns.
Use only black or blue pen to put X marks on the answersheet. Do not use any other
m
ink or pencil.
o m

o
.c.c
aa
Useful Physical lConstants
eel
m
Mass of Earth
ME
m
d
d
Radius of Earth
RE
aa
o
Mass of Sun
M

l o
Radius of Sun
R
nnl
Speed of Light
c
ww
o
Astronomical Unit
1 A. U.
o
d
d
.
Gravitational Constant .
G
ww
Gravitational Acceleration
g
Speed of Sound w
(at
cs
wroom temperature in air)
Specific Heatw
of water
Cw
w

5.97 1024 kg
6.4 106 m
1.99 1030 kg
7 108 m
3 108 m/s
1.5 1011 m/s
6.67 1011 m3 /(Kg s2 )
9.8 m/s2
340 m/s
4.186 103 J/kg oC

Space for Rough Work

for more papers please visit www.downloadmela.com

INAO Sr 2008

Part A: Multiple Choice Questions


1. The dimensions of Boltzmann constant (k) are
(a) M1 L2 T1

(b) M1 L2 T1

(c) M2 L1 T1

(d) M1 L2 T2

Note: Objections are raised over this question, because temperature dimensions are
not specified. Thus, Question was removed from evaluation.
2. P. One can see absorption lines in the Solar spectrum.
Q. The core of the Sun has temperature of more than 1 million degree Celsius and the
Solar surface has temperature of about 6000 degree Celsius.
m
Which of the following options is correct?
o m

c o
a
lisalcorrect.
(b) Statement P is incorrect but Q
ee
(c) Both the statements arem
correct
and Q is the correct reason of
m
d
P.
aad
oocorrect and Q is not the reason of P.
(d) Both the statements lare
nnl
When you stand on the ground,
ww what is the distance of the horizon from you?
oo (c) 15 km (d) 50 km
(a) 500 km (b) 5 km
d
. .d
A regular barometer
ww is thrown from the top of a building.If the barometer is freely
falling, what w
will be the height of the mercury column?
w
ww (b) 76 cm (c) 50 cm (d) 0 cm
(a) 100 cm
. .c
(a) Statement P is correct but Q is incorrect.

3.

4.

5. P. Eclipses are not distributed evenly throughout the year, but happens only in certain
months of a given year.
Q. Orbit of the Moon (around the Earth) makes an angle of roughly 5 degrees to the
orbit of Earth (around Sun).
Which of the following options is correct?
(a) Statement P is correct but Q is incorrect.
(b) Statement P is incorrect but Q is correct.
(c) Both the statements are correct and Q is the correct reason of
P.
(d) Both the statements are correct and Q is not the reason of P.
6. For the Earth, if the perihelion were 147 million km, approximately what will be the
aphelion for the Earth?
Aphelion: Point farthest from the Sun in the orbit of a body about the Sun.
Perihelion: Point nearest from the Sun in the orbit of a body about the Sun.
(a) About 2 times the Perihelion, 300 million km

for more papers please visit www.downloadmela.com

INAO Sr 2008

(b) About 3 times the Perihelion, 450 million km


(c) Slightly more than the perihelion, about 155 million km
(d) Exactly the same as the perihelion, 147 million km
7. When the ball at the end of the string swings to its lowest point, the string is cut by
a sharp knife as shown. Assuming no air resistance, what will be the path of the ball?
Simple Pendulum

String

8.

m
oomA
.c.c
a
B
l la
e
e
Heavy Ball
m
C
ddm
D
aa
lo
lo
(a) A (b) B (c) C (d)nn
D
w
The answer is (c).
oow
.d
If we ever make contact
.dwith aliens, which of our fundamental units is likely to match
theirs? (In otherw
words,
w which of these units is universally fundamental?)
w
wLight year (c) a.m.u. (Atomic Mass Unit) (d) None of these
(a) Kelvin (b)
ww

9. P. In a dynamo, it is necessary that coil should be moving and magnet should be


stationary and not vice versa.
Q. Magnetic force is perpendicular to the direction of motion of charges.
Which of the following options is correct?
(a) Statement P is correct but Q is incorrect.
(b) Statement P is incorrect but Q is correct.
(c) Both the statements are correct and Q is the correct reason of P.
(d) Both the statements are correct and Q is not the reason of P.
10. If a person beats drum on the Earth and an astronaut beats an identical drum in space,
what will be the differences in the effects?
(a) There will be no vibration in the drum in space.
(b) There will be vibration in space but no sound.
(c) The drum on Earth will vibrate for a longer time then the one in space.
(d) There will be no difference in terms of the vibrations or sound.
11. An Olympiad student fails to get a medal. The angry team leader launches him into a
rocket far into space & then throws him out. The student just grazes the upper edge
of the atmosphere during his trajectory. He will...
2

for more papers please visit www.downloadmela.com

INAO Sr 2008

(a) Go around the Earth


(b) Crash on the Moon
(c) Get lost into deep space
(d) Reach the Olympiad venue again
12. P: Gravitational force exerted by Saturn on a human being is approximately same as
that exerted by another human being standing a few cm away.
Q. Saturn has very low density.
(Additional data: Mass of Saturn = 5 1026 kg, Distance of Saturn = 1.4 109 km)
(a) Statement P is correct but Q is incorrect.
(b) Statement P is incorrect but Q is correct.

m
o
cc Q is not the reason of P.
(d) Both the statements are correct .and
.
a
l la
ee process.
P. Temperature is not constant in an adiabatic
Q. Adiabatic processes do not obey ideal
gas equation.
m
m
d
Which of the following options is correct?
aad
(a) Statement P is correct
but Q is incorrect.
lo
lo
n
n but Q is correct.
(b) Statement P is incorrect
ww
(c) Both the statements
oo are correct and Q is the correct reason of P.
d
. .d
(d) Both the statements
are correct and Q is not the reason of P.
ww
A star is seenw
rising from Calcutta (23.5 N 92 E) at 7:00 pm IST, at what time IST
w

will it be seen
wwto rise from Mumbai (19 N 72 E)?
mcorrect reason of P.
(c) Both the statements are correct and Qo
is the

13.

14.

(a) 5:40 pm

(b) 7:00 pm

(c) 7:20 pm

(d) 8:20 pm

15. A broom with a long handle balances at its centre of gravity as shown in the figure. If
you cut the broom into two parts through the centre of gravity and then weigh each
part, which part will weigh more?

(a) The part with the broom will weigh more.


(b) The part without the broom will weigh more.
(c) Both the parts will weigh the same.
(d) It wold depend on the weight of the broom.
16. Star A has temperature 4000 K and star B has temperature 40, 000 K. If the two
stars have roughly same radii, which of the following statements is not true?
(a) B is more luminous than A.
3

for more papers please visit www.downloadmela.com

INAO Sr 2008

(b) A emits more in IR than in UV.


(c) B emits more in UV than in IR.
(d) A emits more in IR than B.
Note: In the actual question the word NOT was missing and the fact that both
stars are assumed to have same radii was not explicitly stated. Thus, the question was
removed from evaluation.
17. Every object exerts gravitational force on every other object - The force exerted by an
object is higher if its mass is higher. Consider 2 magnets - a bigger magnet P and a
smaller one Q. Which of the following will be true?
(a) Magnet P will exert a greater magnetic force than Q.

m
(b) The magnetic forces exerted by P and
Q will be the same.
m
oo

(c) Magnet Q will exert a greater magnetic


.ccforce than P.

a .
ee
Three balls are thrown from the top of
cliff along paths P, Q and R. If their initial
m
m
d
speeds are the same and there is noaairdresistance, under what conditions will the balls
a
strike the ground below with theosame
l lo speed?
nn
P
ww
oo
d
. .d
ww
Q
ww
R
ww
l la and magnetism are unrelated.
(d) We cannot tell from the sizes, as gravity

18.

(a) This will happen if the mass of each ball is the same.
(b) This will happen if the distance traveled by each ball is the same.
(c) This cannot happen unless the paths of the balls are identical.
(d) This will always happen - No additional condition is required.
19. A battery is connected by wires to a bulb as shown below and the bulb glows.
Through which points does the charge flow?

Battery

Lighted
Bulb
3

(a) 1-2-3-4-1. Charge flows through the battery also.


(b) 1-2-3-4. Charge flows through the wires and bulb only.
(c) 2-3.Charge flows only through the bulb.
4

for more papers please visit www.downloadmela.com

INAO Sr 2008

(d) There is no flow of the charge in the circuit.


20. Which of the following is true?
(a) cos 80 = sin 10
(b) cos 120 = cos 240
(c) sin 135 = sin 270
(d) sin 330 = sin 210
The answer is (d).

m
oom
.c.c
a
l a
eel
m
ddm
aa
o
l o
nnl
w
oow
.d
.d
ww
ww
ww

for more papers please visit www.downloadmela.com

INAO Sr 2008

Part B: Analytical Questions


21. A year in Solar calendar consist of 365.25 days and the same in Lunar calendar consist
of 354 days. The additional days in Solar calendar are kept as balance every year.
Whenever the number of balance days exceeds 30, an additional month of 30 days is
added to the lunar year to offset the difference. The cycle goes on. Anwesh, whose
birthday falls on 1st January, noticed that in the year 2008, his birthday coincided
with the start of the lunar year. In which earliest future year, his birthday will again
coincide with the start of the lunar year?
Solution: Every year the Solar year lags by 11.25 days. After 4 years, number of
intercalary day will be integer, i.e. 45.
m
Intercalary days are compensated by a month whenever
oom they exceed 30 days.
Thus, one has to find L.C.M. of 45 and 30. .cc
.
L.C.M. is 90. i.e. after 90 intercalary days
lalaare introduced, both calenders will
ee
match.
i.e. they will match after 8 years. m
ddm
Thus, his birthday in 2016 will again
mark start of the lunar calander.

22.

aa
o
l o
Important: Brute force method
nnl should not be given more than 7 marks.
w
oow
ddthe circuit diagram, given the conditions below:
Find R1 , R2 and R3.in
.
ww
A
ww
ww
R
R
2

R3

(a) If P is connected to A & Q is connected to C then the current in the circuit


is 2A.
(b) If P is connected to A & Q is connected to B then the current in the circuit
is 4A.
(c) If P is connected to C & Q is connected to B then the current in the circuit
is 3A.

for more papers please visit www.downloadmela.com

INAO Sr 2008

Solution: For a given V,


V

(R1 + R3 ) R2
(R1 + R2 + R3 )
(R2 + R3 ) R1
= 4
(R1 + R2 + R3 )
(R1 + R2 ) R3
= 3
(R1 + R2 + R3 )
= 2

(1)
(2)
(3)

Thus,
4(R2 + R3 ) R1 = 2(R1 + R3 ) R2
2R1 R2 = 2R2 R3 4R1 R3
m
o3om
R1 R2 = R2 R
2R1 R3

(4)
(5)

.c.c
a
a 1 + R2 ) R3
4(R2 + R3 ) R1 e=l l3(R
e
R1 m
R3 = 3R2 R3 4R1 R2
m
d1d
R
R
aa 3 = 3R2 R3 4R2 R3 + 8R1 R3
oR2 R3 = 7R1 R3
lo
l
nn
R2 = 7R1
w
oow
2(R1 + R3 ) R2 = 3(R1 + R2 ) R3
.d
.d
R2 R3 = 2R1 R2 3R1 R3
ww
7R1 R3 = 14R12 3R1 R3
ww
ww
10R1 R3 = 14R12
R3 =
The resistances are in ratio
(R1 + R2 + R3 ) =
R3 =
R3 =
R1 =
R1 =
R2 =
R2 =

7
R1
5

R1 : R 2 : R 3 = 1 : 7 :

(6)
(7)
(8)

(9)
(10)
(11)

(12)
7
5

47
R1
5
47
V R1
V (R1 + R2 + R3 )
= 5
3(R1 + R2 )
3 8R1
47
V
120
5
47

V
7 120
47
V
168
47
7
V
168
47
V
24

(13)
(14)

(15)

(16)

(17)

Note: If one finds ratio of resistance correctly, 8.5 marks out of 10 should be
awarded.

for more papers please visit www.downloadmela.com

INAO Sr 2008

23. You are given two lenses of focal lengths f1 and f2 respectively.
(a) Is it possible to arrange them in such a way that both incident beam and
emergent beam of light will be parallel beams? Show all (at least 3)possible
cases (for different lens combinations) with ray diagrams.
(b) Are the incident and emergent beams parallel to each other?
(c) By observing the ray diagrams, state the condition on the distance d between the two lenses in terms of f1 and f2 .
(d) For what combinations of lenses, the said arrangement is not possible?
Solution:

1
1
d
1
m
+
om
=
f
f1 f2 f1 fo
2

.cc

d is the distance between the two lenses. a .


l a
1
If the incident beam as well as the emerging
eel beam are parallel beams, then f = 0
which gives the condition, d = f1 + fm
2
If one lens is concave and other convex,
ddmit will become, d = f1 f2
Thus, it is only possible if focal a
length
a of the convex lens is more than that of the
o
o
l
concave lens.
l
If both lenses are convex, dnis
nalways positive. Hence it is always possible.
wwit is not possible at all.
If both lenses are concave,

oo
d
Note: The ray diagrams
. .d should be such that above relations could be inferred by
measuring repective
ww lengths on the ray diagrams.
ww
The twow
beams
w will be parallel to each other if both lenses are parallel to each
other. Knowledge of the first equation is not expected. Students should infer the
d = f1 + f2 relation by purely observing ray diagrams.

for more papers please visit www.downloadmela.com

INAO Sr 2008

24. If Aniket starts drawing a straight line with a brand new typical ball-point pen, how
long line can be drawn before he finishes his refill? Explicitly state all the assumptions
you make.
Solution: A typical ball pen refill has length of 12 cm.
(Acceptable 10-15 cm)
Typical refill diameter is 1 mm.
(Acceptable 0.5 - 2 mm)
Hence Total volume of ink is r 2 h = (0.05)2 12 cc
V 0.1 cc
Typical thickness of writing is of the size of finite number of molecules.
Size of one ink molecule can be taken to be 0.5 - 1nm.
m
m
Hence the thickness would be roughly 10nm. oo
.c.c
(Acceptable 0.5-50 nm)
a
Typical width is half of refill diameter. l la
ee
Thus, length,
2m
r h
0.05 12
V
= d m
=
cm
l=
t d a2td
r
2 10 107
a
l 9.5 km.
loo

nnl
ww
Note: Answer is notoimportant
in this order of magnitude
o
Approach to the problem
should be judged for marks.
.d
.d
ww
ww
ww

estimation question.

for more papers please visit www.downloadmela.com

Download ebooks at http://www.downloadmela.com/ebooks/ebookdisplay.html


Download Video tutorials at http://www.downloadmela.com/videotutorials/videotutorials.html
Download placement papers,interview questions,certification dumps and many more at
http://www.downloadmela.com/companypapers/itpapers.html m

oom
Download Previous year papers at
.c.c
a
http://www.downloadmela.com/previouspapers/previouspapers.html
l a
eel
m
ddm
Download Magazines at
aa
http://www.downloadmela.com/magazines/magazines.html
o
l o
nnl
w
oow
.d
.d
ww
ww
ww

for more papers please visit www.downloadmela.com

Indian National Astronomy Olympiad 2009

A-PDF Watermark DEMO: Purchase from www.A-PDF.com to remove the watermark

Junior Category

Roll Number:

Roll Number

Model Solutions
INAO 2009
Duration: Three Hours

Date: 31st January 2009


Maximum Marks: 100

Please Note:
Please write your roll number on top of this page in the space provided.
Before starting, please ensure that you have received a copy of the question paper
containing total 3 pages (6 sides).
In Section A, there are 10 multiple choice questionsm
with 4 alternatives out of which
m and -1 mark for each
ooanswer
only 1 is correct. You get 3 marks for each correct
.c.c
wrong answer.

lala
In Section B, there are 4 multiple choiceequestions
with 4 alternatives each, out of
e
which any number of alternatives may
be correct. You get 5 marks for each correct
m
m
answer. No marks are deducted fordd
any wrong answers. You will get credit for
aa mark all correct choices and no wrong
the question if and only ifoyou
l lo
choices. There is no partial
nncredit.
For both these sections,w
you have to indicate the answers on the page 2 of the
oow
answersheet by putting
a
dd in the appropriate box against the relevant question
.
number, like this: .
w
Q.NO. (a) w
Q.NO. (a) (b) (c) (d)
ww(b) (c) (d)
22 w

w   OR 35  
Marking a cross () means affirmative response (selecting the particular choice).
Do not use ticks or any other signs to mark the correct answers.

In Section C, there are 5 analytical questions totalling 50 marks.


Blank spaces are provided in the question paper for the rough work. No rough work
should be done on the answer-sheet.
No calculators are allowed.
The answer-sheet must be returned to the invigilator. You can take this
question booklet back with you.

for more papers please visit www.downloadmela.com

Useful Physical Constants


Mass of the Earth
Radius of the Earth
Mass of the Sun
Radius of the Sun
Radius of the Moon
Speed of Light
Astronomical Unit
Gravitational Constant
Gravitational Acceleration on the Earth
Gravitational Acceleration on the Moon

ME
RE
M
R
Rm
c
1 A. U.
G
g
gm

m
oom
.c.c
Space for RoughaWork
l a
eel
m
ddm
aa
o
l o
nnl
w
oow
.d
.d
ww
ww
ww

5.97 1024 kg
6.4 106 m
1.99 1030 kg
7 108 m
1.7 106 m
3 108 m/s
1.5 1011 m
6.67 1011 m3 /(Kg s2 )
9.8 m/s2
1.6 m/s2

for more papers please visit www.downloadmela.com

INAO Jr 2009

Section A: (10 questions 3 marks each)

1. If ax = by = cz and b2 = ac, then y = ?

2xz
xz
(a) x+z
(b) x+z
(c) 2xz (d) xz
Solution:

a = bx
y
c = bz
y y
b2 = ac = b x b z
y
y
= bx+z
m
y y om
2 =
+
o
x z.cc
y(xla
+a
z).
=
eexzl
2xz
y d=m
m
d z+x

2.

aa
o
Ans = (a)
l o
nnl
ww
o
Each of the figures below,
odepict a constellation.
d
.d
.
ww
ww
ww
(a)

(b)

(c)

Find the odd one out.

(d)

Solution: The constellations are (a) Leo (b) Taurus (c) Scorpio (d) Canis Major.
First three are zodiac signs whereas the fourth one is not.
Ans = (d)
3. Gravitational force between two identical uniform solid gold spheres of radius r each
in contact is proportional to
(a) r 4

(b) r 2

(c)

1
r2

(d)

1
r3

for more papers please visit www.downloadmela.com

INAO Jr 2009

Solution: The distance between two spheres = 2r and masses are the same
F =

GM 2
(2r)2
3

G( 4r
)
3
=
4r 2
6
r
F 2
r
F r4

Thus ans = (a)

4.

m
oom
.c.c
A copper cube and a wooden cube of samela
size
a are initially at room temperature.

l
Then they are kept in an enclosure at 50 e
c.eWhat can we conclude about the temperatures attained by both cubes afterm
5 hours?
m
d
d
(a) Tcopper > Twood as thermal
conductivity
of copper is greater than that of
aa
wood.
lo
lo
n
(b) Twood > Tcopper as specific
heat capacity of wood is greater than that of
n
w
copper.
oow
(c) The temperatures
d will depend on the interplay between specific heat ca.d
.
pacity w
and thermal conductivity of the materials.
w
(d) Both
temperatures will be practically the same, as they are in the enclosure
ww
for
5 hours.
ww
Solution: Both copper and wooden cube will have same temperature, as that
of the enclosure, because the time is sufficiently long to bring them in thermal
equilibrium with their surroundings.
Ans = (d)

5. If the product of all the numbers from 1 to 100 is divisible by 2n , then what is the
maximum possible value for n?
(a) 128

(b) 97

(c) 64

(d) 87

Solution: There are 50 numbers between 1 to 100, which are divisible by at least
first power of 2.
There are 25 numbers, which are divisible by at least second power of 2, i.e. 4.
However, as they are already counted once in previous step, we count them again
for only single power of 2 in this step.
Continuing in the same fashion, there are 12 numbers, which are divisible by at
least third power of 2, i.e. 8.

for more papers please visit www.downloadmela.com

INAO Jr 2009

There are 6 numbers, which are divisible by at least forth power of 2, i.e. 16.
There are 3 numbers, which are divisible by at least fifth power of 2, i.e. 32.
There is only 1 number divisible by sixth power of 2, i.e. 64.
Summing,
(50 + 25 + 12 + 6 + 3 + 1) = 97 is the number of times factor 2 appears in the
product.
So 297 is the highest power of 2, which will be a factor of the product of all the
numbers from 1to 100.
Ans = (b)

mleaking at the rate of 5ml per


6. A repairman on the T. V. tower finds his water bottle
oom
second. He drops the bottle and it reaches the ground
cc straight. If he was at a height
.
of 125m at that time and there was 200ml ofawater
. left in the bottle, the amount of
a
l
water left in the bottle (neglecting air resistance)
e l just before it hit the ground was
e
m
m
(a) 175ml (b) 50ml (c) 100ml dd
(d) 200ml
aa
o
l lo
n
n
Solution: Ans = (d).
w
w and water in it, is the gravitational force (constant
The force acting on aobottle
o
acceleration g). Therefore
because of this free fall motion, bottle and water
.dd
containing in it, will. have same velocity.
w hitting the ground, water bottle will contain the same amount,
Hence just beforew
wwin it.
200ml of water
w
Note: In w
the actual question paper, there was a typographic error with height
of tower specified as 125km rather than 125m. However, as the solution is independent of height, answer doesnt change.
7. In which of the following cities, your shadow will be the shortest, on the 15th of June?
(a) Delhi

(b) Bhopal

(c) Bangalore

(d) Thiruvanantpuram

Solution: The latitude of Bhopal is closest to the Tropic of Cancer. On the


15th June (summer solstice 21st June), the Sun will be almost at zenith and
therefore there we can see our shortest Shadow.
Ans = (b).
8. In the following figure, A, B, C are three light source positions with respect to the obstacle and the screen. Which of these light source positions will result in the longest
shadow of the obstacle on the screen?

for more papers please visit www.downloadmela.com

INAO Jr 2009
11
00
A

11
B00

Screen

11
00
C

Obstacle

.
(a) A
(b) C

(c) A and C form shadows of same length, while


B forms a smaller shadow.
oom

c
l a
eel
m
Solution: Ans = (d)
ddm
aa
o
A
l o
nnl
w
B
oow
C
Obstacle
.d
.d
ww
ww
ww

. .c of same length.
(d) All the three light sources will formashadows

11
00
00
11

K
P

11
00
11
00

Screen
Q

.
From the above figure, P R is the shadow because of light source C and QS is the
shadow because of the light source A.
Now line AK is perpendicular to screen from A, which meets line MN at L.
Since MN k KS,
AMN and AQS are congruent ,
MN
AL
which means
=
.
QS
AK
Similarly, CMN & CP R are congruent,
MN
AL
which means
=
.
PR
AK
= QS=PR.
Therefore, shadows of the obstacle formed by either light source A or C are the
same. In the same manner, we can prove that the shadow formed by the light
source B is also of the same length.

for more papers please visit www.downloadmela.com

INAO Jr 2009

9. Which of the following represents the correct speed-time graph, for a ball bouncing
frequently from a fixed surface?
u

u
t

(a)

(b)

(c)

(d) None.

Solution: Ans = (d).


From the kinematical equation of motion v = (u + at) , we can say that the
speed-time graph should be a linear one. So options
(a) and (c) are not correct.
m
m
o
Also since speed can not assume negative values, even
cco option (b) is incorrect. In
.
fact, (b) represents correct velocity-time graph.in case of inelastic collisions.

10.

lala
ee
Two glass tubes filled with water are m
held
mvertical and connected by a plastic tube
d
d
as shown in the figure. Pans are mounted
on top of each piston such that
aa
(weight of the piston + pan)A l=o(weight
of
the piston + pan)B
o
l
radius of the piston A = 1.0cm
nnand radius of piston B = 1.5cm.
A 30.0gm of mass is added
in
ww pan B, what is the mass required in pan A to balance
o
30.0gm in pan B?
o
d
.d
.
ww
pan
ww
piston
ww
0000
1111
111
000
1111
0000
111
000
1111
0000
111
000
111
000
111
000
A
B
1111
0000
111
000
1111
0000
111
000
111
000
1111
0000
1111
0000
0
11111
11111
00000
0000
111
000
water
1111
0000
111
000
11
00
11
00
1
0
1
0
11
00
11
00
11
00
1
0
1
0
11
00
1
0
11
00
1
0
1
0
1
0
1
0
11
00
1
0
1 00
0
11
11
00
1
0

(a) 67.5gm

(b) 30.0gm

(c) 13.3gm

(d) 24.0gm

Solution: Ans = (c)


Let, mass in pan A = m1 , mass in pan B = m2 = 30gm.
radius of the piston A = r1 = 1.0cm and radius of piston B = r2 = 1.5cm.

for more papers please visit www.downloadmela.com

INAO Jr 2009

Now pressure in both tubes should be the same.


F1
F1
=
A1
A2
m1 g
m2 g
=
r12
r22
m1
30
=
1
1.52
30
m1 =
2.25
30
m1 = 9

P =

m
m

oo
m1 = 13.3gm.
4

11.

.c.c
a
Hence pan A required 13.3gm of mass to balance
l a pan B.
eel
m
ddm
aa
o
l lo
Section B: (4nquestions
5 marks each)
n
ww
Which of the following observations
support the statement that Every system tends
o
to adjust by itself to d
haveominimum Potential Energy.
. .d
(a) Andromeda
ww galaxy and Milky Way galaxy are approaching each other.
(b) Twow
unlike, free charges move towards each other.
w
ww work is required to compress a spring.
(c) External
(d) A powerful magnet can deflect a compass needle from equilibrium position.
Solution: Ans = (b), (c) and (d).
Andromeda Galaxy and the Milky Way are moving around the centre of our local
group of galaxies. In the course of this motion, they just happen to be coming
closer to each other. Their mutual gravitational attraction does not play any
significant role in this motion.

12. In one of the truly revolutionary finds of the 20th century, Howard Carter discovered
discovered tomb of Egyptian Pharaoh (emperor) Tutankhamen in 1922. Along with
the mummy following items were also removed from the tomb. Which of these items
could have been carbon dated to fix the period of the Pharaoh?
(a) Fragments of glass
(b) Bronze Razor
(c) Dried Fruits
(d) Leather Shoe

for more papers please visit www.downloadmela.com

INAO Jr 2009

Solution: Carbon dating relies on the fact that all living objects have 12 C and
14
C in a fixed ratio to each other. The radioactive 14 C keeps decaying to 12 C,
however, it is replaced by food intake consumed by living organisms. After death,
the decay process continues, but there is no replacement for decayed 14 C. Thus
the ratio starts changing.
In the list above, the first two are not made from any organic / living substances.
Thus, they cannot be used for carbon dating.
Ans = (c) and (d).
13. There is a regular bus service between Pune and Mumbai (180km apart) at every
m
hour from both the ends, from 4am to 11pm. These
busses run at average speed
m
o
o
of 45km/hr. Taxies also run on the same route c
at 60km/hr with regular interval of
. are
.c based upon the number of taxies
30min from 5am to 10pm. Following statements
a
a
l
or busses crossed (not overtaken) only during
eel travelling i.e. excluding instances of
arrival and departure. Select the correct statement(s).

m
ddm
aa
o
(b) Last taxi crosses 5 buses.
l lo
n
(c) Bus left at 8pm crosses
n 10 taxis.
ww
(d) Taxi left at 12
noon crosses 6 buses.
oo
d
. .d
ww
Solution: Ans =
ww (a), (b), (c) and (d)
Bus covers the distance in 4 hours, taxi in 3 hours.
ww is at 4am and it will reach its destination at 8am.
First ST bus
(a) First bus crosses 6 taxis.

Thus, during the


journey, it will meet all taxis to have started from the other city from 5 am till
7:30 am. i.e. 5:00, 5:30, 6:00, 6:30, 7:00, 7:30 = 6 taxis
Last taxi starts at 10:00 pm, by which 6:00 pm bus would have arrived at the bus
station. It reaches well past 11:00 pm (time of last bus). Thus, it will meet all
the buses after the 6:00 pm bus. i.e. 7:00, 8:00, 9:00, 10:00, 11:00 = 5 busses.
The taxi leaving at 5:00 pm would have arrived by 8:00 pm. Thus bus will meet
all taxis from 5:30 pm till the last taxi which leaves at 10:00pm i.e. 5:30, 6:00,
6:30, 7:00, 7:30, 8:00, 8:30, 9:00, 9:30, 10:00= 10 taxis
The 8:00 am bus would have arrived by noon. Thus the taxi meets 9:00am,
10:00am, 11:00am, 12:00noon, 1:00pm, 2:00pm buses on the way = 6 buses

14. Which of the following statement(s) is(are) useful, in estimating distances in the
Universe?
(a) Some time Venus can be seen transiting over the solar disc.

for more papers please visit www.downloadmela.com

INAO Jr 2009

(b) Stars with no proper motion appear to change their position in the sky
when viewed six months apart.
(c) Stars exhibit Doppler shift.
(d) All supernovae of Type Ia, have same absolute brightness.
Solution: With Doppler shift we can estimate the velocity of stars but not the
distance.
The Earth-Sun distance was successfully estimated for the first time using Venus
transit method. Option (b) talks of parallax method. The absolute magnitudes
of Supernovae is useful standard candle for cosmological distances.
Hence, ans= (a), (b) and (d).
m

oom
.c.c
a
l a
eel
m
Section C: Analytical
Questions
ddm
aa
(8 marks) What will be area of o
the
largest cyclic quadrilateral that can be inscribed
o
l
in a given circle? Justify your answer
qualitatively (formal proof not necessary).
nnl
ww
o
Solution: Divide any
quadrilateral ABCD, inscribed in a circle, into two trianddo
.
.
gles ABC and ADC,
as shown.
ww
ww
B
ww
h1

P1

C
P2

h2

The area of the triangle is given by the formula,


1
2

base height.

The common base of the two triangles ABC and ADC is given by AC.
The total area, of the quadrilateral will be sum of the areas of the two triangles
i.e.,
1
2

l(AC) (h1 + h2 )

for more papers please visit www.downloadmela.com

INAO Jr 2009

To maximize the total area, l(AC) as well as (h1 + h2 ) should be maximized.


Now the maximum possible length that can be fitted inside a circle has to be its
diameter, d.
Consequently, the total area of the quadrilateral ABCD would be maximum if,
(h1 + h2 ) = d and l(AC) = d
Thus, the maximum area can be 12 d2 .
Noting that the the base and height are perpendicular to each other, it is clear
that the said quadrilateral is a square.
If correct area with no justification: 2 marks.

m
If correct area with incorrect justification: o
3m
marks.

cco
.
If justification only considers cyclic rectangles:
4 marks.
.
lala
ee resulting in wrong area: 7 marks.
If correct justification, but minor error
m
ddm
Any correct method will receive
aa full consideration.
lo
loshe saw a solar eclipse when the size of the solar
(12 marks) Jayshree claimednthat
n
ww
disk was 26 and that of the
lunar disk was 30 . She also claimed that at the time of the
oo between the centres of the two disks was 7. Qualitatively
maximum eclipse, distance
d
show that she could. not
.d have observed a total eclipse. Find the percentage
 of the
1 1
w
solar disk covered w
at the time of the maximum eclipse. (Given: cos
0.49
26
ww
rad).
ww
Solution: At the time of the maximum eclipse, the centres were 7 away from
each other. However, the radius of the solar disk is smaller than that of the lunar
disk by just 2 . Thus, she must not have viewed the total solar eclipse. Let us
find percentage of the maximum partial eclipse.
(1 mark)

p
q
E

B
C t

(1 mark)

for more papers please visit www.downloadmela.com

INAO Jr 2009

In the figure above, points A and B make an angle of 1 and 2 with centre of the
Solar and the Lunar disks (i.e. C and D) respectively.
rs denotes the solar radius, rm the lunar radius and h the separation between the
two centres at the closest approach (all in arcminutes).
[ = rs2
A(ACB)
p+q =

1
2

rs2 1
.
2

2
\ = rm
A(ADB)

2
2m

2 o
rm
2
q + r + t = .cc.o
2.

lala
where r = A(ACD) and t = A(BCD)
ee
Also from figure, r = t.
m
m
d
2a d
2
r 1 r 2
(2 marks)
p = s om a+ 2A(ACD).
2 l lo
2
nn
Now using Heros formula
for area of a triangle,
ww
oo p
A(ACD)
d
. .d = s(s rm )(s rs )(s h)
where,
w
w
ww rs = 13
ww
rm = 15

h = 7
rs + rm + h
s =
2
 
13 + 15 + 7
35
=
s =

2
2
r
35 (35 30) (35 26) (35 14)
A(ACD) =

2
2
2
r2
35 5 9 21

=
2
2 2
2
s
2
573
=
3
4
105
=
3
4
(173.2 + 8.7)

4
91

(2 marks)
2

Now to find 1 and 2 draw a perpendicular line BE to line CD.

10

for more papers please visit www.downloadmela.com

INAO Jr 2009

Let EC = x and EB = y.
In ECB,
x2 + y 2 = rs2 .
and in EDB,
2
(x + h)2 + y 2 = rm
2
2hx + h2 + rs2 = rm
r 2 rs2 h2
x = m
= 0.5
2h
0.5
1
x
=
=
cos1 =
rs
13 m
26
m
1 = 0.49coo
(2 marks)
. .c
Similarlly,
a
a
l
x +l h
0.5 + 7
1
=
cos2 = ee
=
rm
15
2
m
m

a
2dd
=
(1 mark)
3
a

loo
n

l angles are double these values.


These are half angles only.n
Total
ww r2 2
1
s

o
po=

.d
.d
ww
ww
ww

2
rm
2 2
+ 2A(ACD)
2
2

91
169 0.49 225 + 2
2

3
91 7
84 75 +
22


637
9+
22
(9 + 28.95)
38 arcmin2
(1.5 marks)

and A(Sun) = rs2


= 169 arcmin2

(0.5 marks)

The amount of solar surface covered by moon at the time of maximum eclipse
p
) 100%
A(Sun)
38
(1
) 100%
169
2.923
(1
) 100%
13
(1 0.22) 100%.
78 %.
(1 mark)

= (1

11

for more papers please visit www.downloadmela.com

INAO Jr 2009

. (8 marks) The famous Indian astronomer, Aryabhata, expressed the value of in


1
what we now know as continuing fractions i.e.. = 3.1416 = a +
where a,
1
b+
1
c+
d
b, c, d are positive integers. Find a, b, c, d.
Solution:
1416
10000
1
m
= 3+
m
10000 oo
.c.c
1416
a
l la1
e
= 3+e
9912 + 88
m
m
d
1416
aad
1
o= 3 +
lo
l
1
nn
7+
1416
w
oow
88
d
.d
1
.
= 3+
ww
1
ww
7+
1408 + 8
ww
88
1
= 3+
1
7+
1
16 +
88
8
1
= 3+
1
7+
1
16 +
11
Therefore a = 3, b = 7, c = 16, d = 11.
3.1416 = 3 +

If attempted to solve polynomial: 1 mark.


If approximate reciprocals found correctly: 6 marks.
If approximate reciprocals found correctly and final answer is tallied back:
full marks.
For each wrong value out of a, b, c, d: 1 mark each.

12

for more papers please visit www.downloadmela.com

INAO Jr 2009

. Mehul performed an experiment to verify Ohms law. He connected following circuit


to measure voltage and current.

|
A
+

V
|

m
m

oo A the ammeter and K is the


Here, R is the unknown resistance, V the voltmeter,
c
. .c
key. He obtained following readings :
lala
e3e 4
V (v)
1
2
5
6
I (mA) 1.40 2.83m
4.26
5.68
7.11
8.54
ddm
aa to represent the data.
(a) (9 marks) Plot appropriate graph
o
(b) (2 marks) Find the value loflo
nn R.
(c) (1 mark) From the graph,
ww what will be the voltage across the resistance when
o
I = 8mA?
o
d
.d
.
w
w
Solution:
ww
(a) Fromw
the given observation table we have plotted voltage (v) versus current
w
(mA) graph.

V (V)

v/s

I (mA)
Xaxis : 1cm = 1mA
Yaxis : 1cm = 1 volt
O(0,0)

Yaxis

7
6
5

V(V)

4
3
2
1
Xaxis
0

10

(mA)

13

for more papers please visit www.downloadmela.com

INAO Jr 2009

Optimum amount of graph paper should be utilized.

(1 mark)

Choice of scale should be convenient.

(1 mark)

Optimum scale for voltage on larger axis: y-axis 2 big sq. = 1V, x-axis
1 big sq. = 1mA
Optimum scale for current on larger axis: y-axis 2 big sq. = 1V, x-axis
2 big sq. = 1mA
Points should be clearly marked.
Chart Title, Axis Titles etc. should be properly
m written.

(1.5 mark)
(1 mark)

oom
The scales and preferably origin should .be
ccclearly specified at the top right
.
corner.
(1 mark)
lala
ee
Points used for finding slope should
be well separated, well marked and
m
m
preferably not from the existing
(1 mark)
d dataset.
aad
Line should be continuous
(1.5 mark)
oand well balanced.
lo
l
nn
8mA point should be marked
on the graph paper (as shown). (1 mark)
ww
oo
d
(b) Hence from the
above
(2 marks)
. .d graph,
ww
(y2 y1 )
slope =
ww
(x2 x1 )
ww
(4.2 0.7)V
R

(6 1)mA
(3.5)

(5 103 )
R 700
(2 mark)

(c) From the graph, for I = 8mA; V= 5.6V

(1 mark)

. If the entire surface of the earth is covered using A4 size (size of your answer sheet)
sheets of paper, what will be the total weight of paper used?
Solution: The total surface area of the Earth

(2 marks)

A = 4R2
2
= 4 (6.4 108 ) cm2
Now normal size of A4 paper
a = (20 30) cm2

14

for more papers please visit www.downloadmela.com

INAO Jr 2009

To cover the whole earth we will require

(3 marks)

4 6.42 1016
papers 82 1014 papers
20 30
Now thickness of a typical 100 page notebook is 1cm (or thickness of question
paper + answer sheet was about 2mm). So thickness of a single paper = 0.01cm
= 0.1mm.
(1 mark)
mass of a single A4 paper :
m = volume density of a paper

m
m

oo
We take, density of a paper density of wood =
0.5 gm/cc
(1 mark)
c
. .c crumpled paper floats on the
(It should be definitely less than water as even
a
a
water)
ll
ee
mass of a paper 600cmm
0.01cm 0.5gm/cm3
m
d
3a
gm d(1 to 5 gm acceptable)
a
w lo
8.1
o 1015 3
l
n
n2.4 1016 gmwt
w
13
oow 2.4 10 kgwt
9
.d
.d 24 10 14Tonne wt
ww
2.4 10 N
ww
ww
2

Total weight of paper used is about 24 Trillion Kgwt.

prepared using LATEX2

(1 mark)

(2 marks)

for more papers please visit www.downloadmela.com

Download ebooks at http://www.downloadmela.com/ebooks/ebookdisplay.html


Download Video tutorials at http://www.downloadmela.com/videotutorials/videotutorials.html
Download placement papers,interview questions,certification dumps and many more at
http://www.downloadmela.com/companypapers/itpapers.html m

oom
Download Previous year papers at
.c.c
a
http://www.downloadmela.com/previouspapers/previouspapers.html
l a
eel
m
ddm
Download Magazines at
aa
http://www.downloadmela.com/magazines/magazines.html
o
l o
nnl
w
oow
.d
.d
ww
ww
ww

for more papers please visit www.downloadmela.com

Indian National Astronomy Olympiad 2009


Senior Category

Roll Number:

Roll Number

Model Solutions

Please Note:

Date: 31st January 2009


Maximum Marks: 100

INAO 2009
Duration: Three Hours

Please write your roll number on top of this page in the space provided.

Before starting, please ensure that you have received a copy of the question paper
containing total 3 pages (6 sides).

CS

In Section A, there are 10 multiple choice questions with 4 alternatives out of which
only 1 is correct. You get 3 marks for each correct answer and -1 mark for each
wrong answer.
In Section B, there are 4 multiple choice questions with 4 alternatives each, out of
which any number of alternatives may be correct. You get 5 marks for each correct
answer. No marks are deducted for any wrong answers. You will get credit for
the question if and only if you mark all correct choices and no wrong
choices. There is no partial credit.
For both these sections, you have to indicate the answers on the page 2 of the
answersheet by putting a in the appropriate box against the relevant question
number, like this:
Q.NO. (a) (b) (c) (d)
Q.NO. (a) (b) (c) (d)
22

  

OR

35

 

HB

Marking a cross () means affirmative response (selecting the particular choice).


Do not use ticks or any other signs to mark the correct answers.

In Section C, there are 5 analytical questions totalling 50 marks.


Blank spaces are provided in the question paper for the rough work. No rough work
should be done on the answer-sheet.
No calculators are allowed.

The answer-sheet must be returned to the invigilator. You can take this
question booklet back with you.

HOMI BHABHA CENTRE FOR SCIENCE EDUCATION


Tata Institute of Fundamental Research
V. N. Purav Marg, Mankhurd, Mumbai, 400 088

Useful Physical Constants


ME
RE
M
R
Rm
c
1 A. U.
G
g
gm

5.97 1024 kg
6.4 106 m
1.99 1030 kg
7 108 m
1.7 106 m
3 108 m/s
1.5 1011 m
6.67 1011 m3 /(Kg s2 )
9.8 m/s2
1.6 m/s2

Mass of the Earth


Radius of the Earth
Mass of the Sun
Radius of the Sun
Radius of the Moon
Speed of Light
Astronomical Unit
Gravitational Constant
Gravitational Acceleration on the Earth
Gravitational Acceleration on the Moon

HB

CS

Space for Rough Work

HOMI BHABHA CENTRE FOR SCIENCE EDUCATION


Tata Institute of Fundamental Research
V. N. Purav Marg, Mankhurd, Mumbai, 400 088

INAO Sr 2009

Section A: (10 Q 3 marks each)

1. If ax = by = cz and b2 = ac, then y = ?

2xz
xz
(a) x+z
(b) x+z
(c) 2xz (d) xz

Solution:
y

Ans = (a)

CS

a = bx
y
c = bz
y y
b2 = ac = b x b z
y
y
= bx+z
y y
+
2 =
x z
y(x + z)
=
xz
2xz
y =
z+x

2. Each of the figures below depict a constellation. Find the odd one out.

(b)

(c)

HB

(a)

(d)

Solution: The constellations are (a) Leo (b) Taurus (c) Scorpio (d) Canis Major.
First three are zodiac signs whereas the fourth one is not.
Ans = (d)

3. Gravitational force between two identical uniform solid gold spheres of radius r each
in contact is proportional to
(a) r 4

(b) r 2

(c)

1
r2

(d)

1
4r 2

INAO Sr 2009

Solution: The distance between two spheres = 2r and masses are the same
GM 2
(2r)2

F =

)
G( 4r
3
=
4r 2
6
r
F 2
r
F r4
Thus ans = (a)

CS

4. A copper cube and a wooden cube of volume 103 m3 each are initially at room
temperature. They are then moved to an enclosure of ambient temperature 50 C.
What can we conclude about the temperatures attained by both cubes after 5 hours?
(a) Tcopper > Twood as thermal conductivity of copper is greater than that of
wood.
(b) Twood > Tcopper as specific heat capacity of wood is greater than that of
copper.
(c) The temperatures will depend on the interplay between specific heat capacity and thermal conductivity of the materials.
(d) Both temperatures will be practically the same, as they are in the enclosure
for 5 hours.

HB

Solution: Both copper and wooden cube will have same temperature, as that
of the enclosure, because the time is sufficiently long to bring them in thermal
equilibrium with their surroundings.
Ans = (d)

5. If the product of all the numbers from 1 to 100 is divisible by 2n , then what is the
maximum possible value for n?
(a) 128

(b) 97

(c) 64

(d) 87

Solution: There are 50 numbers between 1 to 100, which are divisible by at least
first power of 2.
There are 25 numbers, which are divisible by at least second power of 2, i.e. 4.
However, as they are already counted once in previous step, we count them again
for only single power of 2 in this step.
Continuing in the same fashion, there are 12 numbers, which are divisible by at
least third power of 2, i.e. 8.

INAO Sr 2009

There are 6 numbers, which are divisible by at least forth power of 2, i.e. 16.
There are 3 numbers, which are divisible by at least fifth power of 2, i.e. 32.
There is only 1 number divisible by sixth power of 2, i.e. 64.
Summing,
(50 + 25 + 12 + 6 + 3 + 1) = 97 is the number of times factor 2 appears in the
product.
So 297 is the highest power of 2, which will be a factor of the product of all the
numbers from 1to 100.
Ans = (b)

6. Two vectors P and Q are acting at a point such that their resultant is perpendicular

|
to Q . If is the angle between P and Q then |P
is given by,
|Q|
(b) sec

(c) cos

(d) sec

CS

(a) cos

Solution: See figure

HB

= 180
Q
= cos
P
= cos(180 )
= cos
P

= sec
Q

7. What will be the approximate period of Chandrayaan moving in an orbit 100 km


above the moons surface?
(a) 57 min

(b) 30 min

(c) 118 min

(d) 79 min

Solution: Let R = radius of the Moon, r = radius of the orbit


Using Keplers Law,
4 2 3
r
GMm
4 2 3
r
=
gR2

T2 =
T2

INAO Sr 2009

Radius of orbit of Chandrayaan from center of the moon is, r = 1.7106 m+100km

CS

4 2
(1.8 106 )3
1.6 (1.7 106 )2
4 10 1.83
106
1.6 1.72
45 1.82
106
1.72
6.7 18
103
17


6.7
6.7 +
103
17
7100sec
118min.

T2 =

is the period of Chandrayaan orbit. The calculation above is a back of the envelope
calculation to estimate the period.
Ans = (c)
8. In the following figure, A, B, C are three light source positions with respect to the obstacle and the screen. Which of these light source positions will result in the longest
shadow of the obstacle on the screen?
11
00
A

00
11

00
B11

HB

Screen

00
11
00
11
C

Obstacle

(a) A

(b) C

(c) A and C form shadows of same length, while B forms a smaller shadow.

(d) All the three light sources will form shadows of same length.

Solution: Ans = (d)

INAO Sr 2009

00
11

00
11
A

K
P

11
00
00
B11

11
00
C

Obstacle

Screen
Q

N
R

CS

From the above figure, P R is the shadow because of light source C and QS is the
shadow because of the light source A.
Now line AK is perpendicular to screen from A, which meets line MN at L.
Since MN k KS,
AMN and AQS are congruent ,
MN
AL
which means
=
.
QS
AK
Similarly, CMN & CP R are congruent,
MN
AL
which means
=
.
PR
AK
= QS=PR.

Therefore, shadows of the obstacle formed by either light source A or C are the
same. In the same manner, we can prove that the shadow formed by the light
source B is also of the same length.

HB

9. The following figure shows skeleton chart of the Orion constellation. Approximate
direction North is marked with the letter ....

(a) A

(b) B

(c) C

(d) D

Solution: The Orions head is northwards.


Ans = (a)

INAO Sr 2009

10. Find the resultant focal length for following system where the radius of curvature is
15cm.

111111111111111111111111
000000000000000000000000
000000000000000000000000
111111111111111111111111
000000000000000000000000
111111111111111111111111
000000000000000000000000
111111111111111111111111
000000000000000000000000
111111111111111111111111
000000000000000000000000
111111111111111111111111
000000000000000000000000
111111111111111111111111
000000000000000000000000
111111111111111111111111
000000000000000000000000
111111111111111111111111
000000000000000000000000
111111111111111111111111
000000000000000000000000
111111111111111111111111
000000000000000000000000
111111111111111111111111
000000000000000000000000
111111111111111111111111
000000000000000000000000
111111111111111111111111
000000000000000000000000
111111111111111111111111
000000000000000000000000
111111111111111111111111
000000000000000000000000
111111111111111111111111
000000000000000000000000
111111111111111111111111
000000000000000000000000
111111111111111111111111
000000000000000000000000
111111111111111111111111
000000000000000000000000
111111111111111111111111
000000000000000000000000
111111111111111111111111
000000000000000000000000
111111111111111111111111
000000000000000000000000
111111111111111111111111
000000000000000000000000
111111111111111111111111
000000000000000000000000
111111111111111111111111
000000000000000000000000
111111111111111111111111
000000000000000000000000
111111111111111111111111

11111111
00000000
00000000
11111111
00000000
11111111
00000000
11111111
00000000
11111111
00000000
11111111
00000000
11111111
00000000
11111111
00000000
11111111
00000000
11111111
00000000
11111111
00000000
11111111
00000000
11111111

glass

(b) 60 cm

(c) 120 cm

air

(d)

CS

(a) 40 cm

water

Solution: Refractive Index : nw =

4
3

; ng =

3
2

9
ng
=
nw
8
na
3
Refractive Index of air relative to water: w na =
=
nw
4
Refractive Index of glass relative to water:w ng =

Using the lens maker formula,


1
1
1
= (n 1)(
+
)
f
R1 R2

focal length for plano-convex glass-in-water lens, where R1 = ; R2 = 15cm is,


( 98 1)
1
1
1
1
= (w ng 1)(
+
)=
=
f1
R1 R2
15
120

HB

f1 = 120 cm

Similarly, focal length for plano-concave air-in-water lens, where R1 = -15cm;


R2 = is,
( 3 1)
1
1
1
1
= (w na 1)(
+
)= 4
=
f2
R1 R2
15
60

f2 = 60 cm

Hence resultant focal length will be


1
1
1
1
1
1
=
+
=(
+ )=
f
f1 f2
120 60
40

f = 40 cm. Ans is (a).

INAO Sr 2009

Section B: (4 Q 5 marks each)

11. Which of the following observations support the statement that Every system tends
to configure itself to have minimum Potential Energy.

(a) Andromeda galaxy and Milky Way are approaching each other.
(b) Two unlike, free charges move towards each other.
(c) External work is required to compress a spring.

(d) Light iron dust moves towards powerful magnet in close vicinity.

CS

Solution: Ans = (b), (c) and (d).


Andromeda Galaxy and the Milky Way are moving around the centre of our local
group of galaxies. In the course of this motion, they just happen to be coming
closer to each other. Their mutual gravitational attraction does not play any
significant role in this motion.

12. Consider a sealed frictionless piston cylinder assembly where the piston mass and
atmospheric pressure above the piston remain constant. A gas in the cylinder is
heated and hence it expands. Which of the following is / are true?
(a) The density of the gas will increase.

(b) The pressure of the gas will decrease.

(c) The internal energy of the system will remain the same.
(d) In this process work is done by the gas.

HB

Solution: As gas in the cylinder is heated and it expands so work is done by


the gas. At the new equilibrium, the density of the gas would have decreased
(same mass in larger volume), internal energy would have increased (proportional
to change in temperature) and the pressure would have remained the same (as
that of atmospheric pressure + pressure due to piston mass).
Ans = (d).

13. In one of the truly revolutionary finds of the 20th century, Howard Carter discovered
the tomb of the Egyptian Pharaoh (emperor) Tutankhamun in 1922. Following items
were removed from the tomb, along with the mummy of the Pharaoh. Which of these
items could have been carbon dated to fix the period of the Pharaoh?
(a) Fragments of glass

(b) Golden Bracelets


(c) Dried Fruits

(d) Leather Shoe

INAO Sr 2009

Solution: Carbon dating relies on the fact that all living objects have 12 C and
C in a fixed ratio to each other. The radioactive 14 C keeps decaying to 12 C,
however, it is replaced by food intake consumed by living organisms. After death,
the decay process continues, but there is no replacement for decayed 14 C. Thus
the ratio starts changing.
In the list above, the first two are not made from any organic / living substances.
Thus, they cannot be used for carbon dating.
Ans = (c) and (d).

14

14. Which of the following phenomena is / are useful, in estimating distances in the
Universe?
(a) Some time Venus can be seen transiting over the solar disc.

CS

(b) Stars with no proper motion appear to change their position in the sky
when viewed six months apart.
(c) Stars exhibit Doppler shift.

(d) All supernovae of Type Ia have same absolute brightness.

HB

Solution: With Doppler shift we can estimate the velocity of stars but not the
distance.
The Earth-Sun distance was successfully estimated for the first time using Venus
transit method. Option (b) talks of parallax method. The absolute magnitudes
of Supernovae is useful standard candle for cosmological distances.
Hence, ans= (a), (b) and (d).

Section C: Analytical Questions

. (8 marks) What will be area of the largest cyclic quadrilateral that can be inscribed
in a given circle? Justify your answer qualitatively (formal proof not necessary).
Solution: Divide any quadrilateral ABCD, inscribed in a circle, into two triangles ABC and ADC, as shown.

INAO Sr 2009

h1

P1

P2

h2

CS

The area of the triangle is given by the formula,


1
2

base height.

The common base of the two triangles ABC and ADC is given by AC.
The total area, of the quadrilateral will be sum of the areas of the two triangles
i.e.,
1
2

l(AC) (h1 + h2 )

To maximize the total area, l(AC) as well as (h1 + h2 ) should be maximized.


Now the maximum possible length that can be fitted inside a circle has to be its
diameter, d.
Consequently, the total area of the quadrilateral ABCD would be maximum if,
(h1 + h2 ) = d and l(AC) = d

HB

Thus, the maximum area can be 21 d2 .


Noting that the the base and height are perpendicular to each other, it is clear
that the said quadrilateral is a square.
If correct area with no justification: 2 marks.
If correct area with incorrect justification: 3 marks.
If justification only considers cyclic rectangles: 4 marks.
If correct justification, but minor error resulting in wrong area: 7 marks.

Any correct method will receive full consideration.

. (12 marks) Jayshree claimed that she saw a solar eclipse when the size of the solar
disk was 26 and that of the lunar disk was 30 . She also claimed that at the time of the
maximum eclipse, distance between the centres of the two disks was 7 . Qualitatively
show that she could not have observed a total eclipse. Find the percentage
 of the
1 1
0.49
solar disk covered at the time of the maximum eclipse. (Given: cos
26
9

INAO Sr 2009

rad).

Solution: At the time of the maximum eclipse, the centres were 7 away from
each other. However, the radius of the solar disk is smaller than that of the lunar
disk by just 2 . Thus, she must not have viewed the total solar eclipse. Let us
find percentage of the maximum partial eclipse.
(1 mark)

C t

CS

(1 mark)

In the figure above, points A and B make an angle of 1 and 2 with centre of the
Solar and the Lunar disks (i.e. C and D) respectively.
rs denotes the solar radius, rm the lunar radius and h the separation between the
two centres at the closest approach (all in arcminutes).

HB

[ = rs2
A(ACB)
p+q =

1
2

rs2 1
.
2

2
\ = rm
A(ADB)

2
2

2
rm
2
.
q+r+t =
2

where r = A(ACD) and t = A(BCD)


Also from figure, r = t.
p=

2
rs2 1 rm
2

+ 2A(ACD).
2
2

10

(2 marks)

INAO Sr 2009

Now using Heros formula for area of a triangle,


A(ACD)
where,
rs
rm
h

s(s rm )(s rs )(s h)

CS

= 13
= 15
= 7
rs + rm + h
s =
2
 
13 + 15 + 7
35
s =

=
2
2
r
35 (35 30) (35 26) (35 14)
A(ACD) =

2
2
2
2
r
35 5 9 21
=

2
2 2
2
s
2
573
3
=
4
105
=
3
4
(173.2 + 8.7)

4
91
(2 marks)

HB

Now to find 1 and 2 draw a perpendicular line BE to line CD.


Let EC = x and EB = y.
In ECB,
x2 + y 2 = rs2 .
and in EDB,
2
(x + h)2 + y 2 = rm
2
2hx + h2 + rs2 = rm
r 2 rs2 h2
x = m
= 0.5
2h
0.5
1
x
=
=
cos1 =
rs
13
26
1 = 0.49
(2 marks)
Similarlly,
x+h
0.5 + 7
1
cos2 =
=
=
rm
15
2

(1 mark)
2 =
3

11

INAO Sr 2009

These are half angles only. Total angles are double these values.

2
2 2
rs2 2 1 rm

+ 2A(ACD)
2
2

91
169 0.49 225 + 2
2
3

91 7
84 75 +
22


637
9+
22
(9 + 28.95)
38 arcmin2
(1.5 marks)

p =

CS

and A(Sun) = rs2


= 169 arcmin2

(0.5 marks)

The amount of solar surface covered by moon at the time of maximum eclipse
p
) 100%
A(Sun)
38
) 100%
(1
169
2.923
) 100%
(1
13
(1 0.22) 100%.
78 %.
(1 mark)

= (1

HB

. (8 marks) The famous Indian astronomer, Aryabhata, expressed the value of in


1
what we now know as continuing fractions i.e.. = 3.1416 = a +
where a,
1
b+
1
c+
d
b, c, d are positive integers. Find a, b, c, d.

12

INAO Sr 2009

Solution:
3.1416 = 3 +

= 3+

1416
10000
1
10000
1416
1
9912 + 88
1416
1
1
7+
1416
88
1
1
7+
1408 + 8
88
1
1
7+
1
16 +
88
8
1
1
7+
1
16 +
11

= 3+

CS

= 3+

= 3+

= 3+

HB

= 3+

Therefore a = 3, b = 7, c = 16, d = 11.

If attempted to solve polynomial: 1 mark.


If approximate reciprocals found correctly: 6 marks.
If approximate reciprocals found correctly and final answer is tallied back:
full marks.
For each wrong value out of a, b, c, d: 1 mark each.

. (10 marks) Kedar sent a container of marbles by road from Mumbai to Parag in
Pune. The container was 2m 2m 2.5m in size, with height being the larger
dimension. Marbles of 2 cm diameter were arranged to fill the entire base and then
additional layers of marbles were arranged with each upper marble exactly on top
of corresponding marble in the previous layer (see figure). The marbles were thus
13

INAO Sr 2009

CS

placed upto 2m height to complete the cubical structure. However, on reaching Pune,
when Parag opened the container, he found the height of structure was not 2m as
promised by Kedar, but something else. Kedar defended saying that marbles may
have readjusted due to jiggling. Can you find new height of this marble pile?

Solution: Total number of marbles is 100100100 = 106 arranged in total 100


layers. The marbles will rearrange themselves so as to occupy vacant spots in the
structure. We assume that there is enough jiggling in the trip from Mumbai to
Pune so that uniformity is attained in the arrangement of the marbles throughout
the box.
There are three ways to solve this problem and all of them lead to nearly the
same answer. (All three methods earn full credit).

HB

Method 1: The marbles will be rearranged in a face-centred cubic structure.


In this structure, the bottom layer has 100100 marbles, arranged such that each
4 marbles create a gap between them, into which a marble can be placed in the
next layer. Since there are 99 99 such gaps, the next layer will consist of 99 99
marbles. The layer above that will again have 100 marbles, and so on. Thus, we
will have 50 layers of 100 100 marbles and 50 layers of 99 99 marbles. But in
addition, we shall have extra marbles displaced from the 99 99 layers. Number
of marbles displaced in each such layer is (1002 992 ) = 199. There are total 50
such layers. Thus, total number of marbles remaining after filling 100 layers is
n = 199 50
= 200 50 50
= (100 100) 50

The 101st layer can be of 100 100 marbles. The number of marbles remaining
are in fact smaller than this number. Thus, there will be one additional incomplete
layer.
In this formation each set of 4 marbles on one layer and the 1 marble on the
next layer sitting in the gap between them form a pyramid. Let the height of
this pyramid calculated from the plane containing the centres of the 4 marbles in

14

INAO Sr 2009

h
s

CS

the first layer to the centre of the top marble be h cm. Since the radius of each
marble is 1 cm, according to the left panel in the figure below,

2 2 s2
h =
p
4 (12 + 12 )
=

2
=

Now,
Distance between floor and centre of marbles in the 1st
layer is 1 cm,
st
nd
Vertical separation between 1 and 2 layer centres is 2 cm,
Vertical separation between 2nd and 3rd layer centres is 2 cm,
and so on.
For 101st layer, distance between the top and marble centres will again be 1 cm.
Thus, total height of the structure will be,

HB

H = 1 + 100 2 + 1 cm
1 + 141.4 + 1 cm
143.4 cm

Method 2: The marbles will be rearranged in a hexagonal close-packed structure. In this structure, the bottom layer has 100 marbles in the first row next to
a wall, but the adjacent row has 99 marbles, fitting into the gaps of the first row.
The next row will have again 100 marbles, and so on. The distance between the
lines connecting the centres of one row of marbles to the next row is (see right
panel of figure above)

22 12 cm = 3 cm.
The distance from the centre of the first row to the wall is 1 cm, and the distance
from the centre of the last row to the opposite wall is 1 cm. Therefore if the
number of rows between opposite walls is r, then

1 + (r 1) 3 + 1 = 200

15

INAO Sr 2009

Solving, we get r = 198/1.73 + 1 115 (r must be rounded off to the nearest


lower integer, since the walls are fixed). Thus one can put 58 rows of 100 marbles
and 57 rows of 99 marbles in the first layer. That is a total of 58 100 + 57 99 =
100 + 57 100 2 57 = 11443 marbles in the first layer.

The second layer can be arranged in exactly the same manner, but now each
row will fit in the gaps created by the adjacent rows in the first layer. Since the
pattern of gaps is reverse of the pattern of marbles in the first layer, one can put
58 rows of 99 marbles and 57 rows of 100 marbles in the second layer. That is a
total of 58 99 + 57 100 = 58 100 2 58 100 = 11442 marbles in the
second layer.

CS

Thus the first two layers will consist of 11443 + 11442 = 22885 marbles. From
the third layer, the pattern will repeat. Therefore, we shall have 106 /22885 44
(rounding off to the nearest larger integer) pairs of layers, i.e., 88 layers in all (the
last layer being incomplete).

HB

In this formation each set of 3 marbles on one layer and the 1 marble on the next
layer sitting in the gap between them form a tetrahedron. Let the height of this
tetrahedron calculated from the plane containing the centres of the 3 marbles in
the first layer to the centre of the top marble be d cm. Since the radius of each
marble is 1 cm, according to the right panel in the figure below,

d =
22 b2
r
2 2
=
4(
2 1 2 )2
3
r
2 2
4(
3)
=
3
r
4
4
=
3

= 2 2/ 3
Now,
Distance between floor and centre of marbles in the 1st layer
is 1 cm,

Vertical separation between 1st and 2nd layer centres is 2 2/ 3 cm,
Vertical separation between 2nd and 3rd layer centres is 2 2/ 3 cm,
and so on.
For 88th layer, distance between the top and marble centres will again be 1 cm.
Thus, total height of the structure will be

H =

87 2 2

1+
+ 1 cm
3
174 1.414
1+
+ 1 cm
1.732
2 + 100 1.414 cm
144 cm

16

INAO Sr 2009

Method 3: The labour involved in the above two solutions can be avoided to
a great extent by recognising that the two arrangements described above are, in
fact, the face-centred cubic and hexagonal close packing arrangements. In
each of these cases, the packing fraction is 0.74, i.e., the actual volume occupied
by the marbles is 74% of the apparent volume of the stack. Therefore, if V is the
volume occupied in each of these arrangements, then V is given by the equation
0.74V = 106

4
(1cm)3
3

(1)

Now, V = A H, where A is the area of the stack. Here, A = 200 cm2 . Thus,

CS

H = V /A
106 4
3
cm
=
0.74 4 104
100 3.1416

cm
3 0.74
100 104.72
cm

74
141.4 cm

We obtain an answer close to those of the first two methods. The small discrepancy among the three methods arises due to the difference in packing of the balls
near the edges, i.e., close to the walls (which manifests itself in the approximations
that we have to make in each case). The answer obtained in the third method is
the ideal case scenario, where we did not consider any edge effect.

HB

(In the last method, recognising that 0.74 is actually


lation.)

3 2

will simplify the calcu-

. Chiraag performed an experiment using a simple pendulum to find value of g. He


measured time taken for 30 oscillations of the pendulum for various values of length
(repeated thrice for each value of length), following readings were obtained.
L
Readings
cm t1 (Sec) t2 (Sec) t3 (Sec)
20.0
26.9
26.9
27.0
25.0
30.1
30.1
30.1
30.0
32.9
32.8
32.7
35.0
35.6
35.8
35.7
40.0
38.0
38.1
38.1
45.0
40.4
40.4
40.5

Mass of the bob of the pendulum was known to be 50 gm.


(a) (9 marks) Plot appropriate graph to represent the data.

(b) (2 marks) Find the value of g.

17

INAO Sr 2009

(c) (1 mark) Using the graph, find out required length of the pendulum to get a
time period of 1.24 sec.

Solution: The equation for period of simple pendulum is given as,


s
L
T = 2
g
T2 =

4 2
L
g

In the experiment above, time is measured for 30 oscillation, So we have to average


out three readings for each length and then find time for single oscillation.
Readings
t1 (Sec) t2 (Sec) t3 (Sec)
26.9
26.9
27.0
30.1
30.1
30.1
32.9
32.8
32.7
35.6
35.8
35.7
38.0
38.1
38.1
40.4
40.4
40.5

tavg

T= 30t

(cm)

20.0
25.0
30.0
35.0
40.0
45.0

T2

(Sec)

(Sec)

Sec2

26.9
30.1
32.8
35.6
38.1
40.5

0.897
1.03
1.09
1.19
1.27
1.35

0.81
1.06
1.19
1.41
1.61
1.82

CS

Data should be properly averaged converted to time period for one oscillation.
(1.5
marks)
Proper choice of variable on x-axis.

(1 mark)

Optimum amount of graph paper should be utilized.

HB

T 2 valuesshould be computed and linear graph should be plotted. (Graph


of T vs. L is also acceptable).
(2 marks)

Choice of scale should be convenient.

(0.5 mark)

Points should be clearly marked.

Chart Title, Axis Titles etc. should be properly written.

(1 mark)
(0.5 marks)

The scales and preferably origin should be clearly specified at the top right
corner.
(0.5 marks)
Points used for finding slope should be well separated, well marked and
preferably not from the existing dataset.
(0.5 marks)
Line should visually appear to be well balanced.
38cm point should be marked on the graph paper (as shown).
marks)

18

(1 mark)
(0.5

INAO Sr 2009

The T 2 Vs. L graph should be plotted to find slope.


2

T Vs. L
2.0

Scale
xaxis: 1unit = 5cm
yaxi: 1unit = 0.2 sec
Origin (0,0)

1.8

1.6

1.52
1.4

1.2

T2 (Sec 2 )

1.0

0.8

0.6

CS

0.4

0.2

38

(0,0)

10

15

20

25

30

35

40

45

L (cm)

From the graph,

(1.61 .79)
(40 20)
(.82)
= 4.1 102

(20)

slope

4 2
4.1 102
g
40
g
4.1 102
g 9.8 m/s2

HB

(2 mark)

Using the graph, when T = 1.24 sec, T 2 = 1.54 sec2 . Thus, L 38 cm.
mark)

prepared using LATEX2

(1

Indian National Astronomy Olympiad 2010


Junior Category

Roll Number:

Roll Number

Question Paper
Date: 30th January 2010
Maximum Marks: 100

INAO 2010
Duration: Three Hours

Please Note:
Please write your roll number on top of this page in the space provided.

Before starting, please ensure that you have received a copy of the question paper
containing total 3 pages (6 sides).

CS

In Section A, there are 10 multiple choice questions with 4 alternatives out of which
only 1 is correct. You get 3 marks for each correct answer and -1 mark for each
wrong answer.
In Section B, there are 4 multiple choice questions with 4 alternatives each, out of
which any number of alternatives may be correct. You get 5 marks for each correct
answer. No marks are deducted for any wrong answers. You will get credit for
the question if and only if you mark all correct choices and no wrong
choices. There is no partial credit.
For both these sections, you have to indicate the answers on the page 2 of the
answer sheet by putting a in the appropriate box against the relevant question
number, like this:
Q.NO. (a) (b) (c) (d)
Q.NO. (a) (b) (c) (d)
22

  

OR

35

 

Marking a cross () means affirmative response (selecting the particular choice).


Do not use ticks or any other signs to mark the correct answers.

HB

In Section C, there are 5 analytical questions totaling 50 marks.


Blank spaces are provided in the question paper for the rough work. No rough work
should be done on the answer-sheet.
No computational aides like calculators, log tables, slide rule etc. are allowed.
The answer-sheet must be returned to the invigilator. You can take this
question booklet back with you.

HOMI BHABHA CENTRE FOR SCIENCE EDUCATION


Tata Institute of Fundamental Research
V. N. Purav Marg, Mankhurd, Mumbai, 400 088

Useful Physical Constants


ME
RE
M
R
Rm
c
1 A. U.
G
cs

5.97 1024 kg
6.4 106 m
1.99 1030 kg
7 108 m
1.7 106 m
3 108 m/s
1.5 1011 m
6.67 1011 m3 /(Kg s2 )
330 m/s
23.5

Mass of the Earth


Radius of the Earth
Mass of the Sun
Radius of the Sun
Radius of the Moon
Speed of Light
Astronomical Unit
Gravitational Constant
Speed of Sound in Air
Inclination of the Earths Axis

HB

CS

Space for Rough Work

HOMI BHABHA CENTRE FOR SCIENCE EDUCATION


Tata Institute of Fundamental Research
V. N. Purav Marg, Mankhurd, Mumbai, 400 088

INAO Jr 2010

Section 1:Multiple Choice Questions


Part A: (10 Q 3 marks each)

1. As seen from the Earth, the stars appear to twinkle, while the planets do not because,
(a) Light coming from the stars gets absorbed by interstellar dust.

(b) the stars are self luminous while the planets merely reflect the light.
(c) Angular sizes of the stars are much smaller than the planets.
(d) All the above.

CS

A wooden cube of length 5 units is painted on all faces and then cut in 85 smaller
cubes of varying sizes. The sides of each of the smaller cubes are some integer number
of units. All bigger cubes are carved out of the corners of the original cube. Answer
the following three questions:
2. How many cubes have no side painted?
(a) 15

(b) 17

(c) 20

(d) 27

HB

Solution: If you cut the bigger cube in smaller cubes with all side of 1 unit,
there will be 125 total cubes.
Each cube of size 2 will fuse 8 such smaller cubes together. i.e. the total number
of cubes will reduce by 7.
Similarly, each cube of size 3 and 4 will reduce total number of cubes by 26 and
63 respectively.
Now, we have 85 total cubes, which is 40 less than 125. This means there is 1
cube of size 3 and 2 cubes of size 2 at three of the corners. Rest all are size 1.
If there were 125 smaller cubes of size 1, one can say the volume which is not
touched by paint on any face is equivalent to a cube of size 3 i.e. 27 smaller cubes.
However, the the bigger cubes carved out of corners include some of these inner
cubes as well. 8 such inner smaller cubes are included in the size 3 cube and 1
each is included in the 2 size 2 cubes.
Thus, number of unpainted cubes = 27 8 1 1 = 17

3. How many cubes have exactly 2 sides painted?


(a) 24

(b) 27

(c) 30

(d) 36

Solution: To have exactly 2 sides painted, the smaller cube should be along any
of the edges but not at the corner. There are total 12 edges to the original cube.
For each edge, there will be 3 cubes satisfying the condition if all cubes were of
size 1. How ever, there is a size 3 cube, which includes 2 such smaller cubes each

INAO Jr 2010

along 3 of the edges.


Similarly, the 2 size 2 cubes will each include 1 such smaller cube each along 3 of
the edges.
Thus, total number of cubes = 12 3 2 3 1 3 1 3 = 24

(a) 69

(b) 64

(c) 62

(d) 50

4. If one was to cut the original cube (of size 5 units each side) with a condition that
all corners would be occupied by the cubes bigger than 1 unit size, what will be the
smallest total of cube pieces possible?

CS

Solution: There are two possibilities which will satisfy this condition. Either all
corners will have cubes of size 2 or else there will be 1 cube of size 3 and rest all
corners will have cubes of size 2.
There are total 8 corners.
In first case total number of cubes will be = 125 8 7 = 69
In second case, total number of cubes will be = 125 7 7 26 = 50
5. In a given watch, the minute and the hour hand come together successively exactly
after 65 minutes. Does the watch gain or lose time and how much per hour?
(a) Gains about 27 seconds.
(b) Loses about 27 seconds.
(c) Neither gains nor loses.

(d) Information insufficient.

HB

Solution: The angular rate of the two hands of the clock should be compared
to find the interval at which the two hands coincide.
2 = m t h t


2 2
=
t

1
12
12
hr
t =
11
60
min
= 1 hr
11
300
= 65 min
sec
11
3
= 65 min 27 sec
11

As the clocks hands are coinciding faster than they ideally should, the clock is
gaining time.

INAO Jr 2010

6. Two parallel wires carrying current in opposite direction will


(a) exert a force twisting the wires.
(b) attract each other.
(c) repel each other.

(d) not exert any force on each other.


7. If you hold a magnifying glass of focal length 10 cm in the sunlight and place a piece
of paper at its focus, you can burn a hole in the paper. What could be the size of
this hole?
(a) 10 mm

(b) 5 mm

(c) 0.5 mm

(d) 0.1 mm

CS

Solution: If the magnification is given by M, Object distance, Object size, Image


Distance, Image size by u, O, v&I respectively,
M =

7 108

v
I
=
u
O
f
500c
0.1 7 108
500 3 108
7
103 mt.
15
0.5 mm

HB

8. During an earthquake, an earthquake monitoring Centre observed that transverse


waves traveling with speed 4.5 km/s arrived at the centre 3 minutes after the longitudinal waves traveling at 8.2 km/s. Deduce the approximate distance to the epicenter.
(a) 60 km

(b) 220 km

(c) 660 km

(d) 1800 km

Solution:

8.2t = 4.5 (t + 180)


3.7t = 810
810
D = 8.2t = 8.2
3.7
= 2.22 810
D 1800km

9. A certain person nicknamed Enthu, encountered an automatic staircase (i.e. escalator) at a shopping complex, which was moving upward at a constant rate. Just for

INAO Jr 2010

the fun of it he decided to walk up this escalator at the rate of one step a second.
Twenty steps brings him to the top. Next day he goes up at two steps a second and
reaching the top in 32 steps. How many steps are there in the escalator?
(a) 40

(b) 60

(c) 64

(d) 80

Solution: Let the escalator rise with the speed of n steps per second.
On first day, Enthu reaches the top after 20 manual steps i.e. in 20 seconds.
On second day, Enthu reaches the top after 32 / 2 = 16 seconds.
Thus, total number of steps,
=
=
=
=
=

16 (n + 2)
32 20 = 12
3
20 (3 + 1)
80

CS

20 (n + 1)
4n
n
Steps

10. Two stars of masses M and 3M respectively are going around each other, inq
near circular orbits, with period T . The separation between them is given by D =
The value of k is,

(a) 0.5

(b) 1

(c) 1.5

kGM T 2
.
2

(d) 3

Solution: The Centre of mass of the system will be located at distance D4 from
the heavier mass. Centripetal force is provided by the mutual gravitation.

HB

G M (3M)
D
= (3M) 2
2
D
4
D 4 2
GM
=
D2
4 T2
GMT 2
D3 =
2
r
2
3 GMT
D =
2

Section B: (4 questions 5 marks each)

11. If we throw a ball in a shallow water tank, propagation velocity of ripples on surface
of the water will depend on
(a) surface tension of the water

(b) depth of the water tank


(c) density of the water

(d) height from which the ball was dropped


4

INAO Jr 2010

Solution: Note: The propagation velocity will vary as per the depth of the water,
provided the water is not too deep. However, the derivation for this effect includes
concepts beyond the level of participants. Thus, the accepted solution for this
question would be a and c must be selected, d must not be selected and b
would be acceptable both ways.
12. Amit decided to experiment with cannon ball by making it hollow and filling water
inside it. He then punched few holes in it. After the ball was fired horizontally, he
was expecting to see water jets coming out from some of the holes. Which of the
following locations of holes will allow water jet to come out?
(b) Back, top and sides

(c) back and bottom

CS

(a) Front, bottom and sides


(d) None of the holes

Solution: Horizontally, all particles (i.e. cannonball and water particles) are
flying with the same velocity. Hence water cannot come out of front / back /side
holes. Vertically, all particles are experiencing free fall. Thus, gravity cannot
exert any additional pressure on water and neither can water be slower than the
cannonball. Hence no water can come out from top or bottom.
13. In the progress of Astronomy over the ages, we find several instances of startling
new observations changing our ideas about the Universe and lead to new theories.
Listed below are some milestones in the history of Astronomy and observations which
necessitated them. Pick the correct statement(s).
(a) Ptolemy was aware of retrograde motion of planets when he gave
his model of the solar system.

HB

(b) Newton was aware of Keplers Laws when he gave his Law of
Gravitation.
(c) Einstein felt need to modify Newtons theory of Gravitation to explain the
expansion of the Universe.

(d) Existence of Cosmic Microwave Background Radiation (CMBR) led to the


creation of the Big Bang Theory.

Solution: Ptolemy needed epicyclic model of the solar system as that was the
only way to fit observed motions of the planets including the retrograde motion.
Keplers laws enabled Newton to deduce r12 nature of the law of gravitation.
Einstein gave General theory of relativity in 1916 and assumed the Universe to
be non-expanding as per then norm. Hubble discovered the expansion of the
Universe in late 20s forcing Einstein to revisit his own theory.
Existence of CMBR was one of the key predictions of the big bang theory. The
theory was formulated before CMBR was discovered.

INAO Jr 2010

14. Two simple pendula are hung close to each other on a thin, rigid support and are
allowed to oscillate independently in planes parallel to each other. They have periods
3 seconds and 7 seconds respectively with same amplitudes and are initially released
from the opposite extreme positions of each other. At which of the following times
the threads of the two pendula will be coplanar again?
(b) 7.88 seconds

(c) 10.50 seconds

(d) 23.10 seconds

(a) 1.05 seconds

Section C: Analytical Questions

. An alien civilisation on a star far far away came to know about the Astronomy
Olympiad Examination and wanted to test smartness of the students. They sent
following two coded secret messages. Decode them.

CS

(a) (5 marks) First is a pictorial message in black and white colour. They sent it
on radio waves in the form of ones and zeros. Find out what the picture says.
00000 00000 00000 00000 00000 00000 11111 01100 00100 00100 00011
10000 01000 10100 01000 10100 01101 10000 10001 00100 10001 11000
10001 00001 00010 00101 00100 01001 10110 01111 10100 00110 10000
01001 11000 00000 00000 00000 00000 00000 000
(b) (5 marks) Surprisingly, the aliens are also proficient in English and the following
coded message is actually a sentence in English language. In the answer sheet,
write down the coded sentence and also the true meaning of each alphabet in
the code.
Up tpmwf uif qsfwjpvt tvcrvftujpo uijol pg uif nfttbhf tfou cz uif
bsfdjcp ufmftdpqf up bmjfot.

HB

Solution: In 1974, a pictographic message was sent to outer space by radio waves
using Arecibo telescope. The total number digits in the message was product of
two prime numbers. It was argued that any intelligent being can rearrange the
grid in a rectangle with sides measuring these two prime numbers and then use
blank square for 0 and filled square for 1 to reveal the picture.

In the present message same idea is used. Total number of digits is 203 which is a
semiprime number; i.e. it can only be divided as 29 7. If the grid is rearranged
in this way, following pattern is seen:









Marking Scheme:

INAO Jr 2010

Realising total number of digits are 203, which is a semi-prime number. (2


marks)
Attempt to draw either 29 7 or 7 29 grid.

(1 mark)
(2 marks)

picking the message INAO.

Part (b) is a substitution cipher.


The word uif appears thrice in the sentence. Obviously, it stands for the.
Looking at the two words, the substitution rule is inferred as every letter is
substituted by the next letter to create the coded message. Deciphering,
To solve the previous subquestion think of the message sent by the arecibo
telescope to aliens. Marking Scheme:

CS

Mapping the alphabets and deciphered Sentence.

(5 marks)

Mapping the alphabets and Deciphered Sentence, but with small syntactic
errors.
(4.5 marks)
Mapping the alphabets only.

(4 marks)

Deciphered Sentence but Alphabet mapping is not given.

(3 marks)

Only partial decipherment with more than 10 letters.

(2 marks)

Only partial decipherment with 7-10 letters.

Only partial decipherment with the deciphered correctly.

(1.5 marks)
(1 mark)

HB

. (8 marks) In the following table, the first column gives various optical phenomena /
instruments and the top row gives various optical effects which may help in explaining
them. In the answers sheet, tick the correct effect(s) involved in each phenomena in
appropriate rows.
Appreciable
Dispersion

Internal
Reflection
Reflection

Blue Sky
Mirage
Rainbow
Smooth Convex
Mirror
Thick Concave
Lens
Solution:

Refraction Scattering

INAO Jr 2010

Int. Refle.

X
X

Refle.

Refra.

Scat.
X

X
X
X

Blue Sky
Mirage
Rainbow
Sm. Conv. Mirror
Th. Conc. Lens

Appri. Disp.

Blue Sky (1 mark): One mark for scattering. -0.5 for each additional tick
mark.
Mirage (2 marks): One each for internal reflection and refraction. 0 marks
if internal reflection is not ticked.

CS

Rainbow (2 marks): One for internal reflection; 0.5 each for dispersion
and refraction. 0 marks if internal reflection is not ticked.
Mirror (1 mark): One mark for reflection. 0 if any additional / other tick
marks.
Thick Lens (2 marks): One each for dispersion and refraction.
Any options not mentioned above, do not carry either positive or negative credit.

HB

. Study the following image of the night sky (bigger version is printed in your answersheet) for mid-night on a certain day at a certain place and answer the questions
below it. All answers must be marked / written on the answer-sheet.

CS

INAO Jr 2010

(a) (2 marks) Mark all the four directions on the map.

(b) (1 mark) Is this place in northern part of India or the southern part?
(c) (3 marks) Mark and name 2 constellations each from the following two lists:

HB

0.5 marks each; any 2: Orion, Ursa Major, Taurus, Leo, Cygnus, Scorpio
1 mark each; any 2: Hydra, Corvus, Aquarius, Cancer, Canis Major,
Aries

(d) (2 marks) Sketch rough band of ecliptic i.e. apparent path of the Sun, the Moon
and all the planets in the sky.
(e) (2 marks) In which month the sky will appear like this at this time? Give reason
in one line.
Solution: (a) From Top clockwise: East, North, West, South. Note the position
of Ursa Minor Constellation (i.e. Pole star) to the right of the map. Further,
Taurus is at the bottom and Leo is at the top.
(0.5 marks per direction)
If the direction pointers are off by small angle (less than 10 ), overall -0.5 marks.
(b) Ursa Minor is very close to Horizon. Hence the latitude of the place is not
very high. Thus, the place is in Southern India.
(-0.5 if wrong reason is
mentioned.)
(c) Cygnus, Scorpio, Aquarius are not on the map. Rest are clearly visible.

INAO Jr 2010

(d) The band of ecliptic should pass through all the zodiac signs (i.e. Aries
to Virgo). Better precision than this is not expected. For every zodiac sign
significantly away from the ecliptic, -0.5 marks (minimum zero).
(e) At mid-night, Cancer is nearly at Zenith. Hence the Sun is roughly 6 zodiac
signs away i.e. in Capricorn. Thus, it will reach Vernal Equinox (in Pisces) in
roughly two months. Hence, current date is about two months before 21st March.
Thus, the current month is January. 0.5 marks for getting month (December February accepted). Reason 1.5 marks.
. Mayank visited a place located at latitude and longitude 82.5 E on 21st June. He
observed that at local noon, shadow of a one meter stick standing vertically on the
ground was 26.8 cm long due south.

CS

(a) (5 marks) Find latitude of the place.

(b) (5 marks) Find the day on which the shadow of this stick at the local noon will
be longest and find length and direction of that shadow.
Note:
sin1 (0.268) = 15.5, cos1 (0.268) = 74.5, tan1 (0.268) = 15.0 , tan(2 )
1
and 3 = 1.73
30
Solution: Shadow is 0.268 m due South on the day of the Summer solstice. This
means the Sun is to the North of Zenith, i.e. the place is south of tropic of cancer
i.e. in southren India.
(1 mark)
On that day, the Sun is directly above the tropic of cancer, i.e. 90 23.5 = 66.5
away from the North Celestial Pole.
(1 mark)
If the altitude of the Sun at the local noon is ,
1
1
=
0.268
tan(15)

= tan(75 )
= 75 66.5
= 8.5 (3 marks)

HB

tan Jun =

In part (b) For a northern hemisphere place, the longest shadow of the stick
will be cast on the Winter Solstice day. The Sun at local noon will be 47 further south than its position on the Summer Solstice day. Hence the Sun will be
180 75 47 = 58 high on the South of the Zenith.
(2 marks)

10

INAO Jr 2010

1
tan 58
= tan 32
= tan(30 + 2 )
tan 30 + tan 2
=
1 tan 30 tan 2
1 + 1
30
3

1
1 13 30

30 + 3

30 3 1
31.73
31.73

51.9 1
51
0.62m

CS

l =

The Shadow will be 0.62 m long due North.

(3 marks)

. Sketch approximate graphs for the following situations:

(a) (4 marks) See the figure below. A tank of water (height of water column b) is
kept on a electronic weighing scale. A metal cube (side a and density ) is hung
from a spring balance and the spring balance is slowly lowered into the tank till
the cube reaches the bottom of the tank. The distance of separation between
the bottom of the tank and bottom of the cube is denoted by h with initial value
h0 .

HB

spring
balance

cube

111111111
000000000
water
000000000
111111111
000000000
tank111111111
b
000000000
111111111
000000000
111111111
a

00.00

weighing scale

Sketch the graph of reading on the spring balance as a function of h.

(b) (3 marks) For the situation above, sketch the graph of reading on the electronic
scale as a function of h.
(c) (2 marks) For the situation above, sketch the graph of sum of the reading on
the electronic scale and the reading on the spring balance as a function of h.

(d) (3 marks) For a typical primary mirror used in Newtonian telescope, sketch a

11

INAO Jr 2010

graph of object distance, u versus image distance, v. All physically measurable


lengths should be taken as positive.
In all cases, mark the significant points on the graph and give their coordinates.

Solution:

w
a 3

a 3

a 3 (1)

CS

(0,0)

a3

(ba)

h0

(0,0)

(ba)

h0 h

w
a 3

2f
f

HB

f 2f

(0,0)

h0

Partial marks given on basis of shape of the curve, neatness of figure, proper
nomencleture (axis titles etc.), proper markingof points and writing their
co-ordinates etc.
In (a), (b) and (c), it is acceptable to take mass of water as m instead of
zero. No points cut.

The y-coordinates can be either in mass units or in force units (multiplication by g). No points cut.
Density of water can be written as w or its CGS values (i.e. 1) or its MKS
value (i.e. 1000). No points cut.

12

INAO Jr 2010

HB

CS

In (d), the curve between u = 0 and u = f may be plotted in both quadrant


1 and quadrant 2. No points cut.

13

INAO Jr 2010

HB

CS

Space for Rough Work

prepared using LATEX2

Indian National Astronomy Olympiad 2010


Senior Category

Roll Number:

Roll Number

Question Paper
Date: 30th January 2010
Maximum Marks: 100

INAO 2010
Duration: Three Hours

Please Note:
Please write your roll number on top of this page in the space provided.

Before starting, please ensure that you have received a copy of the question paper
containing total 3 pages (6 sides).

CS

In Section A, there are 10 multiple choice questions with 4 alternatives out of which
only 1 is correct. You get 3 marks for each correct answer and -1 mark for each
wrong answer.
In Section B, there are 4 multiple choice questions with 4 alternatives each, out of
which any number of alternatives may be correct. You get 5 marks for each correct
answer. No marks are deducted for any wrong answers. You will get credit for
the question if and only if you mark all correct choices and no wrong
choices. There is no partial credit.
For both these sections, you have to indicate the answers on the page 2 of the
answer sheet by putting a in the appropriate box against the relevant question
number, like this:
Q.NO. (a) (b) (c) (d)
Q.NO. (a) (b) (c) (d)
22

  

OR

35

 

Marking a cross () means affirmative response (selecting the particular choice).


Do not use ticks or any other signs to mark the correct answers.

HB

In Section C, there are 5 analytical questions totaling 50 marks.


Blank spaces are provided in the question paper for the rough work. No rough work
should be done on the answer-sheet.
No computational aides like calculators, log tables, slide rule etc. are allowed.
The answer-sheet must be returned to the invigilator. You can take this
question booklet back with you.

HOMI BHABHA CENTRE FOR SCIENCE EDUCATION


Tata Institute of Fundamental Research
V. N. Purav Marg, Mankhurd, Mumbai, 400 088

Useful Physical Constants


ME
RE
M
R
Rm
c
1 A. U.
G
cs

5.97 1024 kg
6.4 106 m
1.99 1030 kg
7 108 m
1.7 106 m
3 108 m/s
1.5 1011 m
6.67 1011 m3 /(Kg s2 )
330 m/s
23.5

Mass of the Earth


Radius of the Earth
Mass of the Sun
Radius of the Sun
Radius of the Moon
Speed of Light
Astronomical Unit
Gravitational Constant
Speed of Sound in Air
Inclination of the Earths Axis

HB

CS

Space for Rough Work

HOMI BHABHA CENTRE FOR SCIENCE EDUCATION


Tata Institute of Fundamental Research
V. N. Purav Marg, Mankhurd, Mumbai, 400 088

INAO Sr 2010

Section 1:Multiple Choice Questions


Part A: (10 Q 3 marks each)

1. As seen from the Earth, the stars appear to twinkle, while the planets do not because,
A. Light coming from the stars gets absorbed by interstellar dust.

B. the stars are self luminous while the planets merely reflect the light.
C. Angular sizes of the stars are much smaller than the planets.
D. All the above.

CS

A wooden cube of length 5 units is painted on all faces and then cut in 85 smaller
cubes of varying sizes. The sides of each of the smaller cubes are some integer number
of units. All bigger cubes are carved out of the corners of the original cube. Answer
the following three questions:
2. How many cubes have no side painted?
A. 15

B. 17

C. 20

D. 27

HB

Solution: If you cut the bigger cube in smaller cubes with all side of 1 unit,
there will be 125 total cubes.
Each cube of size 2 will fuse 8 such smaller cubes together. i.e. the total number
of cubes will reduce by 7.
Similarly, each cube of size 3 and 4 will reduce total number of cubes by 26 and
63 respectively.
Now, we have 85 total cubes, which is 40 less than 125. This means there is 1
cube of size 3 and 2 cubes of size 2 at three of the corners. Rest all are size 1.
If there were 125 smaller cubes of size 1, one can say the volume which is not
touched by paint on any face is equivalent to a cube of size 3 i.e. 27 smaller cubes.
However, the the bigger cubes carved out of corners include some of these inner
cubes as well. 8 such inner smaller cubes are included in the size 3 cube and 1
each is included in the 2 size 2 cubes.
Thus, number of unpainted cubes = 27 8 1 1 = 17

3. How many cubes have exactly 2 sides painted?


A. 24

B. 27

C. 30

D. 36

Solution: To have exactly 2 sides painted, the smaller cube should be along any
of the edges but not at the corner. There are total 12 edges to the original cube.
For each edge, there will be 3 cubes satisfying the condition if all cubes were of
size 1. How ever, there is a size 3 cube, which includes 2 such smaller cubes each

INAO Sr 2010

along 3 of the edges.


Similarly, the 2 size 2 cubes will each include 1 such smaller cube each along 3 of
the edges.
Thus, total number of cubes = 12 3 2 3 1 3 1 3 = 24

A. 69

B. 64

C. 62

D. 50

4. If one was to cut the original cube (of size 5 units each side) with a condition that
all corners would be occupied by the cubes bigger than 1 unit size, what will be the
smallest total of cube pieces possible?

CS

Solution: There are two possibilities which will satisfy this condition. Either all
corners will have cubes of size 2 or else there will be 1 cube of size 3 and rest all
corners will have cubes of size 2.
There are total 8 corners.
In first case total number of cubes will be = 125 8 7 = 69
In second case, total number of cubes will be = 125 7 7 26 = 50
5. It is easier to balance on a faster bicycle moving along a straight line than a slower
one because of
A. conservation of linear momentum

B. conservation of angular momentum

C. conservation of linear momentum and angular momentum


D. conservation of energy

HB

6. On a nice sunny day in Chennai, Akshay saw a supersonic fighter plane flying parallel
to the ground in the sky. As a student of Aeronautical Engineering, he knew off-hand
the speed of the plane to be 1.25 Mach (i.e. 1.25 times the speed of sound in air). He
could hear its sonic boom, 12 seconds after the plane flew directly overhead. What
is the altitude of the plane?
A. 3.7 Km

B. 4.0 Km

C. 5.3 Km

D. 6.6 Km

Solution: Half angle of the cone of the sonic boom is given by sin =
vp is velocity of the plane and vs is the speed of the sound in air.

vs
,
vp

where

INAO Sr 2010

vs
= 0.8
1.25vs
0.6
sin
h
=
cos
vp t
h
1.25 330 12
4 15 330
3
6600 mt. = 6.6 km

sin =
cos =

0.8
=
0.6
h =
=

tan =

A. 10 mm

CS

7. If you hold a magnifying glass of focal length 10 cm in the sunlight and place a piece
of paper at its focus, you can burn a hole in the paper. What could be the size of
this hole?
B. 5 mm

C. 0.5 mm

D. 0.1 mm

Solution: If the magnification is given by M, Object distance, Object size, Image


Distance, Image size by u, O, v&I respectively,
M =

7 108

HB

v
I
=
u
O
f
500c
0.1 7 108
500 3 108
7
103 mt.
15
0.5 mm

8. During an earthquake, an earthquake monitoring Centre observed that transverse


waves traveling with speed 4.5 km/s arrived at the centre 3 minutes after the longitudinal waves traveling at 8.2 km/s. Deduce the approximate distance to the epicenter.
A. 60 km

B. 220 km

C. 660 km

D. 1800 km

INAO Sr 2010

Solution:

8.2t = 4.5 (t + 180)


3.7t = 810
810
D = 8.2t = 8.2
3.7
= 2.22 810
D 1800km

A. 40

B. 60

CS

9. A certain person nicknamed Enthu, encountered an automatic staircase (i.e. escalator) at a shopping complex, which was moving upward at a constant rate. Just for
the fun of it he decided to walk up this escalator at the rate of one step a second.
Twenty steps brings him to the top. Next day he goes up at two steps a second and
reaching the top in 32 steps. How many steps are there in the escalator?
C. 64

D. 80

Solution: Let the escalator rise with the speed of n steps per second.
On first day, Enthu reaches the top after 20 manual steps i.e. in 20 seconds.
On second day, Enthu reaches the top after 32 / 2 = 16 seconds.
Thus, total number of steps,
=
=
=
=
=

HB

20 (n + 1)
4n
n
Steps

16 (n + 2)
32 20 = 12
3
20 (3 + 1)
80

10. Two stars of masses M and 3M respectively are going around each other, inq
near circular orbits, with period T . The separation between them is given by D =
The value of k is,

A. 0.5

B. 1

C. 1.5

kGM T 2
.
2

D. 3

Solution: The Centre of mass of the system will be located at distance

D
4

from

INAO Sr 2010

the heavier mass. Centripetal force is provided by the mutual gravitation.

D 2
G M (3M)
=
(3M)

D2
4
D 4 2
GM
=
D2
4 T2
GMT 2
D3 =
2
r
2
3 GMT
D =
2

Section B: (4 questions 5 marks each)

A.
B.
C.
D.

CS

11. If we throw a ball in a shallow water tank, propagation velocity of ripples on surface
of the water will depend on
surface tension of the water
depth of the water tank
density of the water
height from which the ball was dropped

Solution: Note: The propagation velocity will vary as per the depth of the water,
provided the water is not too deep. However, the derivation for this effect includes
concepts beyond the level of participants. Thus, the accepted solution for this
question would be a and c must be selected, d must not be selected and b
would be acceptable both ways.

HB

12. Amit decided to experiment with cannon ball by making it hollow and filling water
inside it. He then punched few holes in it. After the ball was fired horizontally, he
was expecting to see water jets coming out from some of the holes. Which of the
following locations of holes will allow water jet to come out?
A. Front, bottom and sides
D. None of the holes

B. Back, top and sides

C. back and bottom

Solution: Horizontally, all particles (i.e. cannonball and water particles) are
flying with the same velocity. Hence water cannot come out of front / back /side
holes. Vertically, all particles are experiencing free fall. Thus, gravity cannot
exert any additional pressure on water and neither can water be slower than the
cannonball. Hence no water can come out from top or bottom.

13. In the progress of Astronomy over the ages, we find several instances of startling
new observations changing our ideas about the Universe and lead to new theories.
Listed below are some milestones in the history of Astronomy and observations which
necessitated them. Pick the correct statement(s).
5

INAO Sr 2010

A. Ptolemy was aware of retrograde motion of planets when he gave


his model of the solar system.
B. Newton was aware of Keplers Laws when he gave his Law of
Gravitation.

C. Einstein felt need to modify Newtons theory of Gravitation to explain the


expansion of the Universe.
D. Existence of Cosmic Microwave Background Radiation (CMBR) led to the
creation of the Big Bang Theory.

CS

Solution: Ptolemy needed epicyclic model of the solar system as that was the
only way to fit observed motions of the planets including the retrograde motion.
Keplers laws enabled Newton to deduce r12 nature of the law of gravitation.
Einstein gave General theory of relativity in 1916 and assumed the Universe to
be non-expanding as per then norm. Hubble discovered the expansion of the
Universe in late 20s forcing Einstein to revisit his own theory.
Existence of CMBR was one of the key predictions of the big bang theory. The
theory was formulated before CMBR was discovered.
14. Two simple pendula are hung close to each other on a thin, rigid support and are
allowed to oscillate independently in planes parallel to each other. They have periods
3 seconds and 7 seconds respectively with same amplitudes and are initially released
from the opposite extreme positions of each other. At which of the following times
the threads of the two pendula will be coplanar again?
A. 1.05 seconds

B. 7.88 seconds

C. 10.50 seconds

D. 23.10 seconds

HB

Section C: Analytical Questions

. An alien civilisation on a star far far away came to know about the Astronomy
Olympiad Examination and wanted to test smartness of the students. They sent
following two coded secret messages. Decode them.
(a) (5 marks) First is a pictorial message in black and white colour. They sent it
on radio waves in the form of ones and zeros. Find out what the picture says.
00000 00000 00000 00000 00000 00000 11111 01100 00100 00100 00011
10000 01000 10100 01000 10100 01101 10000 10001 00100 10001 11000
10001 00001 00010 00101 00100 01001 10110 01111 10100 00110 10000
01001 11000 00000 00000 00000 00000 00000 000

(b) (5 marks) Surprisingly, the aliens are also proficient in English and the following
coded message is actually a sentence in English language. In the answer sheet,
write down the coded sentence and also the true meaning of each alphabet in
the code.
Up tpmwf uif qsfwjpvt tvcrvftujpo uijol pg uif nfttbhf tfou cz uif
bsfdjcp ufmftdpqf up bmjfot.

INAO Sr 2010

Solution: In 1974, a pictographic message was sent to outer space by radio waves
using Arecibo telescope. The total number digits in the message was product of
two prime numbers. It was argued that any intelligent being can rearrange the
grid in a rectangle with sides measuring these two prime numbers and then use
blank square for 0 and filled square for 1 to reveal the picture.

In the present message same idea is used. Total number of digits is 203 which is a
semiprime number; i.e. it can only be divided as 29 7. If the grid is rearranged
in this way, following pattern is seen:

CS








Marking Scheme:

Realising total number of digits are 203, which is a semi-prime number. (2


marks)
Attempt to draw either 29 7 or 7 29 grid.
picking the message INAO.

(1 mark)
(2 marks)

HB

Part (b) is a substitution cipher.


The word uif appears thrice in the sentence. Obviously, it stands for the.
Looking at the two words, the substitution rule is inferred as every letter is
substituted by the next letter to create the coded message. Deciphering,
To solve the previous subquestion think of the message sent by the arecibo
telescope to aliens. Marking Scheme:
Mapping the alphabets and deciphered Sentence.

(5 marks)

Mapping the alphabets and Deciphered Sentence, but with small syntactic
errors.
(4.5 marks)
Mapping the alphabets only.

(4 marks)

Deciphered Sentence but Alphabet mapping is not given.

(3 marks)

Only partial decipherment with more than 10 letters.

(2 marks)

Only partial decipherment with 7-10 letters.


Only partial decipherment with the deciphered correctly.

(1.5 marks)
(1 mark)

INAO Sr 2010

. (8 marks) In the following table, the first column gives various optical phenomena /
instruments and the top row gives various optical effects which may help in explaining
them. In the answers sheet, tick the correct effect(s) involved in each phenomena in
appropriate rows.
Appreciable
Dispersion

Internal
Reflection
Reflection

CS

Solution:

Blue Sky
Mirage
Rainbow
Smooth Convex
Mirror
Thick Concave
Lens

Refraction Scattering

Blue Sky
Mirage
Rainbow
Sm. Conv. Mirror
Th. Conc. Lens

Appri. Disp.

Int. Refle.

X
X

Refle.

Refra.

Scat.
X

X
X

Blue Sky (1 mark): One mark for scattering. -0.5 for each additional tick
mark.

HB

Mirage (2 marks): One each for internal reflection and refraction. 0 marks
if internal reflection is not ticked.
Rainbow (2 marks): One for internal reflection; 0.5 each for dispersion
and refraction. 0 marks if internal reflection is not ticked.
Mirror (1 mark): One mark for reflection. 0 if any additional / other tick
marks.

Thick Lens (2 marks): One each for dispersion and refraction.

Any options not mentioned above, do not carry either positive or negative credit.

. Study the following image of the night sky (bigger version is printed in your answersheet) for mid-night on a certain day at a certain place and answer the questions
below it. All answers must be marked / written on the answer-sheet.

CS

INAO Sr 2010

(a) (2 marks) Mark all the four directions on the map.

(b) (1 mark) Is this place in northern part of India or the southern part?
(c) (3 marks) Mark and name 2 constellations each from the following two lists:

HB

0.5 marks each; any 2: Orion, Ursa Major, Taurus, Leo, Cygnus, Scorpio
1 mark each; any 2: Hydra, Corvus, Aquarius, Cancer, Canis Major,
Aries

(d) (2 marks) Sketch rough band of ecliptic i.e. apparent path of the Sun, the Moon
and all the planets in the sky.
(e) (2 marks) In which month the sky will appear like this at this time? Give reason
in one line.
Solution: (a) From Top clockwise: East, North, West, South. Note the position
of Ursa Minor Constellation (i.e. Pole star) to the right of the map. Further,
Taurus is at the bottom and Leo is at the top.
(0.5 marks per direction)
If the direction pointers are off by small angle (less than 10 ), overall -0.5 marks.
(b) Ursa Minor is very close to Horizon. Hence the latitude of the place is not
very high. Thus, the place is in Southern India.
(-0.5 if wrong reason is
mentioned.)
(c) Cygnus, Scorpio, Aquarius are not on the map. Rest are clearly visible.

INAO Sr 2010

(d) The band of ecliptic should pass through all the zodiac signs (i.e. Aries
to Virgo). Better precision than this is not expected. For every zodiac sign
significantly away from the ecliptic, -0.5 marks (minimum zero).
(e) At mid-night, Cancer is nearly at Zenith. Hence the Sun is roughly 6 zodiac
signs away i.e. in Capricorn. Thus, it will reach Vernal Equinox (in Pisces) in
roughly two months. Hence, current date is about two months before 21st March.
Thus, the current month is January. 0.5 marks for getting month (December February accepted). Reason 1.5 marks.
. Mayank visited a place located at latitude and longitude 82.5 E on 21st June. He
observed that at local noon, shadow of a one meter stick standing vertically on the
ground was 26.8 cm long due south.

CS

(a) (5 marks) Find latitude of the place.

(b) (5 marks) Find the day on which the shadow of this stick at the local noon will
be longest and find length and direction of that shadow.
Note:
sin1 (0.268) = 15.5, cos1 (0.268) = 74.5, tan1 (0.268) = 15.0 , tan(2 )
1
and 3 = 1.73
30
Solution: Shadow is 0.268 m due South on the day of the Summer solstice. This
means the Sun is to the North of Zenith, i.e. the place is south of tropic of cancer
i.e. in southren India.
(1 mark)
On that day, the Sun is directly above the tropic of cancer, i.e. 90 23.5 = 66.5
away from the North Celestial Pole.
(1 mark)
If the altitude of the Sun at the local noon is ,
1
1
=
0.268
tan(15)

= tan(75 )
= 75 66.5
= 8.5 (3 marks)

HB

tan Jun =

In part (b) For a northern hemisphere place, the longest shadow of the stick
will be cast on the Winter Solstice day. The Sun at local noon will be 47 further south than its position on the Summer Solstice day. Hence the Sun will be
180 75 47 = 58 high on the South of the Zenith.
(2 marks)

10

INAO Sr 2010

1
tan 58
= tan 32
= tan(30 + 2 )
tan 30 + tan 2
=
1 tan 30 tan 2
1 + 1
30
3

1
1 13 30

30 + 3

30 3 1
31.73
31.73

51.9 1
51
0.62m

CS

l =

The Shadow will be 0.62 m long due North.

(3 marks)

. Sketch approximate graphs for the following situations:

(a) (4 marks) See the figure below. A tank of water (height of water column b) is
kept on a electronic weighing scale. A metal cube (side a and density ) is hung
from a spring balance and the spring balance is slowly lowered into the tank till
the cube reaches the bottom of the tank. The distance of separation between
the bottom of the tank and bottom of the cube is denoted by h with initial value
h0 .

HB

spring
balance

cube

111111111
000000000
water
000000000
111111111
000000000
tank111111111
b
000000000
111111111
000000000
111111111
a

00.00

weighing scale

Sketch the graph of reading on the spring balance as a function of h.

(b) (3 marks) For the situation above, sketch the graph of reading on the electronic
scale as a function of h.
(c) (2 marks) For the situation above, sketch the graph of sum of the reading on
the electronic scale and the reading on the spring balance as a function of h.

(d) (3 marks) For a typical primary mirror used in Newtonian telescope, sketch a

11

INAO Sr 2010

graph of object distance, u versus image distance, v. All physically measurable


lengths should be taken as positive.
In all cases, mark the significant points on the graph and give their coordinates.

Solution:

w
a 3

a 3

a 3 (1)

CS

(0,0)

a3

(ba)

h0

(0,0)

(ba)

h0 h

w
a 3

2f
f

HB

f 2f

(0,0)

h0

Partial marks given on basis of shape of the curve, neatness of figure, proper
nomencleture (axis titles etc.), proper markingof points and writing their
co-ordinates etc.
In (a), (b) and (c), it is acceptable to take mass of water as m instead of
zero. No points cut.

The y-coordinates can be either in mass units or in force units (multiplication by g). No points cut.
Density of water can be written as w or its CGS values (i.e. 1) or its MKS
value (i.e. 1000). No points cut.

12

INAO Sr 2010

HB

CS

In (d), the curve between u = 0 and u = f may be plotted in both quadrant


1 and quadrant 2. No points cut.

13

INAO Sr 2010

HB

CS

Space for Rough Work

prepared using LATEX2

Indian National Astronomy Olympiad 2011


Question Paper

Roll Number:

Roll Number

Date: 29th January 2011


Maximum Marks: 100

INAO 2011
Duration: Three Hours

Please Note:
Please write your roll number on top of this page in the space provided.

Before starting, please ensure that you have received a copy of the question paper
containing total 4 pages (8 sides).

CS

In Section A, there are 10 multiple choice questions with 4 alternatives out of which
only 1 is correct. You get 3 marks for each correct answer and -1 mark for each
wrong answer.
In Section B, there are 4 multiple choice questions with 4 alternatives each, out of
which any number of alternatives may be correct. You get 5 marks for each correct
answer. No marks are deducted for any wrong answers. You will get credit for
the question if and only if you mark all correct choices and no wrong
choices. There is no partial credit.
For both these sections, you have to indicate the answers on the page 2 of the
answer sheet by putting a in the appropriate box against the relevant question
number, like this:
Q.NO. (a) (b) (c) (d)
Q.NO. (a) (b) (c) (d)
22

  

OR

35

 

Marking a cross () means affirmative response (selecting the particular choice).


Do not use ticks or any other signs to mark the correct answers.

HB

In Section C, there are 6 analytical questions totaling 50 marks.


Blank spaces are provided in the question paper for the rough work. No rough work
should be done on the answer-sheet.

No computational aides like calculators, log tables, slide rule etc. are allowed.
The answer-sheet must be returned to the invigilator. You can take this
question booklet back with you.

HOMI BHABHA CENTRE FOR SCIENCE EDUCATION


Tata Institute of Fundamental Research
V. N. Purav Marg, Mankhurd, Mumbai, 400 088

Useful Physical Constants


RE
M
R
c
G

g
h
= 2
Na
MH

6.4 106 m
2 1030 kg
7 108 m
3 108 m/s
6.67 1011 m3 /(Kg s2 )
23.5
10 m/s2
1034 J.s
6.023 1023 mol1
1.008 a.m.u.

Radius of the Earth


Mass of the Sun
Radius of the Sun
Speed of Light
Gravitational Constant
Inclination of the Earths Axis
Gravitational acceleration
Reduced Planck constant
Avogadro constant
Atomic mass of Hydrogen

HB

CS

Space for Rough Work

HOMI BHABHA CENTRE FOR SCIENCE EDUCATION


Tata Institute of Fundamental Research
V. N. Purav Marg, Mankhurd, Mumbai, 400 088

INAO 2011

Section 1:Multiple Choice Questions


Part A: (10 Q 3 marks each)

A. 63 min

C. 127 min

D. 171 min

CS

Solution:

B. 109 min

1. On one starry evening, Nidhi was trying to spot an artificial polar satellite from her
backyard. Typical altitude of any polar satellite is about 800km above surface of the
earth. What is the typical duration after sunset for which Nidhi should try her luck?

HB

Height of the orbit of polar satellite is 800 Km. After Sunset, the Sunlight will
reach satellite for a time (/), where is the angular velocity of the earth and
is as shown in the diagram.
= 15 deg/hr


R
1
= cos
R+h


6.4 106
1
= cos
6.4 106 + 8 105
 
8
= cos1
= cos1 (0.889)
9
!
3
= 30
. cos1 (0.866) = cos1
2
Thus the angle is slightly less than 30 . Thus for this angle, time will be slightly
less than 2 hours.

2. What will be the difference in potential energy (U) of an object of mass M, if it


is lifted from the ground to a height of 2R, where R is the radius of the earth?
2GM
2GM
GM
GM
A.
B.
C.
D.
R
3R
2R
3R
1

INAO 2011

Solution: Let U1 be the potential energy of the object on the ground, U2 be the
potential energy at the height 2R and U be the change in the potential energy


GM
U1 =

R


GM
U2 =

3R
U = U2 U1
 


GM
GM
+
U =
3R
R


2GM
U =
3R

CS

Note: Implicit assumption is that the mass is scaled in earth mass units. Since
the assumption was not explicitly stated, the question was deemed ambiguious
and was dropped from evaluation.
3. Pole star appears stationary because.....

A. Earth is not moving with respect to the pole star.


B. Earth is on the axis of rotation of the pole star.

C. Both Earth and the pole star have same velocity in the Milky Way galaxy.
D. None of the above.

HB

Solution: Pole star appears stationary because it is almost along the axis of
rotation of the earth.
4. Consider a system of two converging lenses, one with focal length of 20cm and the
other with focal length of 5cm, kept 50cm apart. An object is kept at 40cm from the
first lens. What can be said about the image formed on the other side of the second
lens?
A. Erect and Real

B. Inverted and Real

C. Erect and Virtual

D. Inverted and Virtual

Solution: If the object is kept at a distance of 2f, image will also form at the
distance of 2f and that will be real and inverted. The distance between two lenses
is arranged in such a way that the image from the first lens forms at 2f of the
second lens.

INAO 2011

CS

Thus again a real image of this image will be formed at distance 2f on the other
side of second lens and it will invert the inverted image. Thus, final image will
be an erect one.

HB

5. Three rings of same dimensions, are dropped at the same time over identical cylindrical magnets as shown below. The inner diameter of each ring is greater than the
diameter of the magnet.

Which of the following correctly describes the order in which the rings P, Q and R
reach the bottom of the respective magnets?
A. They arrive in the order P, Q, R

B. They arrive in the order P, R, Q


C. Rings P and R arrive simultaneously, followed by Q.

D. Rings Q and R arrive simultaneously, followed by P.

Solution: Plastic is not a conducting material so its motion will not be affected.
When ring Q will drop over the magnet, due to mutual induction eddy currents
will form which oppose the downward motion of copper ring so it will take longer
time to reach at bottom of magnet.
Since R is not complete circular ring, circuit can not be completed but small local
loops of eddy current still form in the ring so it will reach to bottom of magnet
after ring P but before ring Q.

INAO 2011

D. None of the above

6. A charged particle with initial velocity V enters a region with a uniform magnetic

field B = Bbi. If it starts moving along the positive X-axis in a helical path such that

the separation between successive loops is constant, what can be inferred about V ?.

A. V =V b
j

B. V = V b
k

C. V =Vyb
j + Vz b
k

CS

Solution: Lorentzs force is given by F =q( V B )

If initial velocity vector is restricted to only the plane perpendicular to B , particle


would have only circular motion since particle is also progressing along X-axis it
must have parallel component along this direction. So initial velocity should be

V = Vxbi + Vyb
j + Vz b
k, where
Vx 6= 0 and at least one out of Vy and Vz 6= 0
7. If P QRS 4 = SRQP , where P, Q, R and S are distinct non-zero digits. what is
value of R?
A. 1

B. 3

C. 5

D. 7

HB

Solution: P should be even and 4P < 10, hence P = 2.


S is 3 or 8 and 4P S
S=8
Q and R are odd and 4Q < 10
Q=1
R=7

8. From the given P-V diagram, find out the total work done by the gas, while going
from state A to state C.

A. Wtot = WAC

B. Wtot = WBC WAB

INAO 2011

C. Wtot = WAB + WBC


D. Wtot = WAB WBC

Solution: From P-V diagram


Work done is = Area under the curve
Total work done by the gas = Wtot = WAB + WBC
The work done would be differnce in the areas under the curve. However, in case
of work done, the negative sign is implicit in WBC .

A. 10

CS

9. Find out the equivalent resistance at AB from given circuit, if R=10

B. 100

C. 5

D. 6

HB

Solution: We can simplify given circuit as follows

Rres =6

10. How many 3 digit prime numbers can be formed, using digits 5, 6 and 7? Repetition
of digits is allowed.
A. 20

B. 4

C. 7

D. 6

Solution: since we want it to be a prime number, last digit must be 7.


Also one can repeat same digits. Hence, there are 9 possibilities:
557, 567, 577, 657, 667, 677, 757, 767, 777
Since 5 + 6 + 7 = 18 , 567 and 657 both numbers are divisible by 3.
Clearly 777 is divisible by 7, so we have to check remaining 6 numbers out of
which 667 is divisible by 23 and 767 is divisible by 13. Remaining 4 are prime
numbers.

INAO 2011

Section B: (4 questions 5 marks each)

CS

11. Four conducting plates A,B,C,D are arranged as shown in the figure. Plates A and
C are connected to the positive terminal of a DC source and the Plates B and D are
connected to the negative terminal of a DC source. A proton is kept, at the centre of
this assembly. If we disturb the proton slightly from its equilibrium position, which
of the following statements will describe the path followed by the proton.

A. If the proton is displaced slightly towards plate A, it will keep moving


towards plate A.
B. If the proton is displaced slightly towards plate D, it will keep
moving towards plate D.
C. If the proton is displaced slightly along diagonal of the assembly between
plates B and C, it will move directly towards plate B.

HB

D. If the proton is displaced slightly towards plate C, it will come


back to its original position.

Solution: The proton plate A and plate C are positively charged, hence if the
proton is displaced towards plate A, then it will get repelled and return to its
equilibrium position i.e. the centre of the assembly.
Conversely, plate B and plate D are negatively charged, so if the proton is displaced towards them it will get attracted.
Now if the proton is displaced diagonally, positive plates will repel it and negative plates will attract it. Thus it will move towards the closer negative plate.
However, the motion will also have a harmonic oscillator component in direction
of positive plates hence the overall motion may not be termed as direct.

12. A solid copper sphere is kept on an insulating stand. A charge given to it gets distributed uniformly on its surface only. Which of the following factors is/are relevant
to this observation?
6

INAO 2011

A. Copper is a conducting material.


B. Shape of the conductor is a sphere.
C. Like charges repel each other.

D. Potential energy of the system is minimum in this configuration.

CS

Solution: In a perfect conductor, like charges are free to take up equilibrium


positions in response to the Coulomb repulsion between them. Sphere being
symmetric, there will not be any accumulation of charges at any point and hence
there will be uniform distribution over the surface of the sphere, This is the
energetically most favorable distribution of the charge. Hence the potential energy
will be minimum in this case. Since the material of the sphere is a good conductor,
all charges will only reside on the outer surface, whether the interior is hollow or
solid.
13. A block of mass 5 kg is initially at rest on a rough horizontal surface having coefficient
of static friction s = 0.5 and coefficient of kinetic friction k = 0.3. A gradually
increasing horizontal force is applied for dragging it. Assuming g = 10 m/s2 , acceleration of the block and dragging force acting on the block could be respectively given
by,
A. 7 m/s2 , 50 N B. 2 m/s2 , 25 N C. 0 m/s2 , 20 N D. 3 m/s2 ,40 N
Solution: Initially the mass is stationary therefore, the frictional force acting on
it is Fs = s mg i.e. 25 N.
if the external horizontal force applied is smaller than Fs , then the acceleration
produced is zero. Hence, choice 3 i.e. 0, 20 is correct.

HB

Similarly, if the applied horizontal force is greater than the frictional force then
the acceleration produced can be calculated as follows:
ma = Fh Fk
For Fh = 50N, a = 7m/s2
For Fh = 40N, a = 5m/s2

exactly
Fh = 25N, we have to consider s and not k .
2
Now
a = at
0m/s

14. Two stars are seen close to each other in the sky. Star A appears brighter than Star
B. Which of the following statements satisfactorily explain the difference in their
observed brightness? Assume both the stars to be perfect black bodies.
A. Both the stars are identical except for the fact that star A is
closer to us than star B.
B. Both the stars are at same distance, but star A appears yellow,
where as star B appears orange.
7

INAO 2011

C. Both the stars are identical except for the fact that star A has smaller
radius than star B.
D. Both the stars are identical except for the fact that star A is less massive
than star B.

CS

Solution: If both the stars are identical in mass and age, and if star A is closer
to us than star B, then its apparent brightness will be greater than star B. Now if
both the stars are at same distance and have same mass, then star A will appear
brighter if it is intrinsically bright i.e. it is hotter (blue is hotter than yellow) than
star B. the size and the mass of the star does not explicitly explain the brightness
of a star. A star B having smaller radius than star A does not specify that star
B will be brighter than star A. Similarly a more massive star need not always be
brighter than a low mass star.

Section C: Analytical Questions


. (8 marks) In the following table, the first column gives the names of various bright
stars in the sky and the top row gives the names of some zodiacal constellations. In
the answers sheet, tick mark the constellation to which they may belong. Wrong
tick marks carry negative points.
Aries

Taurus

Gemini

HB

Star Name
Aldebaran
Antares
Castor
Denebola
Hamal
Pollux
Regulus
Sirius
Spica

Leo

Virgo Scorpio

Solution:

Star Name
Aldebaran
Antares
Castor
Denebola
Hamal
Regulus
Sirius
Spica
Pollux

Aries

Taurus
X

Gemini

Leo

Virgo Scorpio
X

X
X

X
X
X

INAO 2011

Marking scheme:
One mark for each correct answer.
-0.5 for each wrong answer.

Sirius should be left blank. No marks for leaving it blank. Negative marks
for putting a tick mark in that row.
If more than one constellations are ticked for same star, it is counted as
wrong answer.
total 8 marks.

CS

. (8 marks) Prof. Subramanium Chandrasekhar was first to suggest that the white
dwarf stars will havean upper
2 limit on their mass, which is given by


1
c b
Mlimit = k a G
e mH
where a, e and k are dimensionless integers, with k 1 and e 2 is called
mean number of nucleons per electron. mH is the mass of one hydrogen atom. This
is famously known as Chandrasekhar Mass Limit for which he won Nobel Prize in
1983. Find a and b.
Solution: b can be found by dimensional analysis as follows (3 marks for
getting correct b)

2
1
1.44M = k a
e mH
!b
J s m/s
1
kg =
N m2
(kg)2
kg 2


HB

c
G

b 

kg m2
kg m
and
J
=
s2
s2

b
m2 kg
s m/s
1
s2

kg =
kgm 2
m
(kg 2 )2
s2

now, N =

kg 2

b = 3/2

Similarly a can be found by substituting the values of all the constants given

INAO 2011

and equating it to 1.44M , which is the famous Chandrasekhar mass limit.


=
=

a
a
a

CS

a
a
a
a

2

1034 3 108 3/2 6.023 1026
)
a(
6.67 1011
2 1.008

36 1052
1
3/2
a(
)

2.22 1015
4
30
1.44 2 10 (2.22 1015 )3/2
9 1052
0.16 2 (2.22)3/2 1022 1022.5
q
(0.32)2 (2.22)3 10

0.1024 10.7 10
10.9
3.47
3

(1mark)

1.44 2 10

30

Marking scheme:

All 4 marks for getting correct answer.

3 marks for close enough integer answers.

2 marks for getting only correct order of magnitude (i.e. 0).


0.5 marks deducted if final answer is not an integer.
+1 mark for overall clarity of solution.

HB

. (8 marks) Hot solar plasma is emitted from surface of a circular sunspot whose diameter is 10,000 km. When the plasma reaches the height of 16,000 km above the surface
of the sun its horizontal cross section is measured to have diameter of 90,000km. Assuming that the edge of the plasma cone is parabolic, find the depth inside the sun
from which the plasma started. Assume that the viscosity and magnetic permeability
remains same inside and outside the solar surface.

10

INAO 2011

Solution:
r2
(3marks)
(r + h)2
r2
=
(r + h)2
r
=
(r + 16000)
= 8000km (3marks)

CS

d1
d2
1

9
1

3
r

(2 marks) Marking scheme:

Wrong parabola (x2 = 4ay) considered. Deduct 4 marks.

HB

Linear terms in y included in the equation, grading as per merit of justification.

. (8 marks) Vinita studied a star for 55 days in succession. She noted down the temperature of the star everyday, which was varying in a nice symmetric manner. The
data of her observations is given below. Help Vinita to find the mean temperature
and the period of temperature variation of this star by any suitable method. Give
proper justification for the method used.

11

INAO 2011

Temperature
5312
5264
5250
5273
5328
5400
5472
5527
5550
5536
5488
5419
5345
5284
5253
5257
5297
5363

HB

Solution:

Days
20
21
22
23
24
25
26
27
28
29
30
31
32
33
34
35
36
37

Days
38
39
40
41
42
43
44
45
46
47
48
49
50
51
52
53
54
55

Temperature
5437
5503
5543
5547
5516
5455
5381
5312
5264
5250
5273
5328
5400
5472
5527
5550
5536
5488

Temperature
5472
5527
5550
5536
5488
5419
5345
5284
5253
5257
5297
5363
5437
5503
5543
5547
5516
5455
5381

CS

Days
1
2
3
4
5
6
7
8
9
10
11
12
13
14
15
16
17
18
19

12

INAO 2011

Maximum values
Minimum values
Days
Temperature
Days
Temperature
3
5550
9
5253
16
5547
22
5250
28
5550
34
5253
41
5547
47
5250
53
5550

CS

after exactly 25 days you get same maximum temperature and there is one peak
in between. so the period is 25/2 = 12.5 days.
from the data maximum 5550
minimum 5250
mean = 5400o C
since data contains some incomplete period, mean of all 55 readings will give
incorrect answer. Since data is symmetric, mean by merely finding peaks and
taking average is reasonably correct.
Marking Scheme:

Period value: 2 marks

Period justification: 1 mark


Mean value: 2 marks

Correct method for finding mean: 3 marks

If graphical method is used, upto 2 marks for correct drawing of graph.

HB

No credit for averaging over all 55 readings.

. (9 marks) On one fine day, Akshay was watching DTH television from Madurai
in Tamil Nadu (78 07 E; 9 48 N). He got a call from his IIT friend, Sujeet, who
was watching DTH television of the same company from Salem in Tamil Nadu
(78 07 E; 11 39 N). Both were getting their DTH signals from the same satellite
located at 36000 km directly above a point at the same longitude but at latitude of
10 43.5 N. Find the angle difference in the antenna pointing for Akshay and Sujeet.

Solution:

13

INAO 2011

CS

(3 marks)
Let x be the angle of the antenna with respect to horizon. Radius of the earth
(r) = 6400 km.
l r d
x 90
r
90
d
1.85
6400

90
36000
2
64
1.85
90

200
36
90 0.32 0.52
1
90
6

HB

The antenna at Salem will be pointing 16 south of local zenith and antenna at

Madurai 61 north of local zenith. The secular angle difference is 13 or 20


(4
marks).
The total angle difference (i.e. difference in angles measured w.r.t. local horizons)
would be 20 + 1 51 = 2 11
(2 marks)

. Sketch the graphs of following functions in the space provided on the answersheet
(Plotting on a graphsheet is not expected):
(a) (3 marks) |x + 1| + |x 1|

(b) (3 marks) x + sin(x).


(c) (3 marks) xlog(x).

14

INAO 2011

HB

CS

Solution:

Criterian
Shape
Marking scheme: Values
Slope
Concept

Graph 1 Graph 2
1
2
1

15

Graph 3
1
2

INAO 2011

HB

CS

Space for Rough Work

prepared using LATEX2

Indian National Astronomy Olympiad 2012


Question Paper

Roll Number:

Roll Number

Date: 28th January 2012


Maximum Marks: 100

INAO 2012
Duration: Three Hours

Please Note:
Please write your roll number on top of this page in the space provided.

Before starting, please ensure that you have received a copy of the question paper
containing total 4 pages (8 sides).

CS

In Section A, there are 10 multiple choice questions with 4 alternatives out of which
only 1 is correct. You get 3 marks for each correct answer and -1 mark for each
wrong answer.
In Section B, there are 2 multiple choice questions with 4 alternatives each, out of
which any number of alternatives may be correct. You get 5 marks for each correct
answer. No marks are deducted for any wrong answers. You will get credit for
the question if and only if you mark all correct choices and no wrong
choices. There is no partial credit.
For both these sections, you have to indicate the answers on the page 2 of the
answer sheet by putting a in the appropriate box against the relevant question
number, like this:
Q.NO. (a) (b) (c) (d)
Q.NO. (a) (b) (c) (d)
22

  

OR

35

 

Marking a cross () means affirmative response (selecting the particular choice).


Do not use ticks or any other signs to mark the correct answers.

HB

In Section C, there are 5 analytical questions totaling 60 marks.


Blank spaces are provided in the question paper for the rough work. No rough work
should be done on the answer-sheet.

No computational aides like calculators, log tables, slide rule etc. are allowed.
The answer-sheet must be returned to the invigilator. You can take this
question booklet back with you.

HOMI BHABHA CENTRE FOR SCIENCE EDUCATION


Tata Institute of Fundamental Research
V. N. Purav Marg, Mankhurd, Mumbai, 400 088

Useful Physical Constants

6 1024 kg
6.4 106 m
2 1030 kg
7 108 m
1.7 106 m
3.84 108 m
3 108 m/s
1.5 1011 m
6.67 1011 m3 /(Kg s2 )
23.5
10 m/s2

ME
RE
M
R
Rm
dm
c
1 A. U.
G

CS

Mass of the Earth


Radius of the Earth
Mass of the Sun
Radius of the Sun
Radius of the Moon
Distance to the Moon
Speed of Light
Astronomical Unit
Gravitational Constant
Inclination of the Earths Axis
Gravitational acceleration

HB

Space for Rough Work

HOMI BHABHA CENTRE FOR SCIENCE EDUCATION


Tata Institute of Fundamental Research
V. N. Purav Marg, Mankhurd, Mumbai, 400 088

INAO 2012

Section 1:Multiple Choice Questions


Part A: (10 Q 3 marks each)

1. If the square of your age in seconds gives the age of the Universe in seconds, then
B. you are in primary school
C. you are a young adult

A. you havent started to walk yet

D. you were born before British left India

CS

Solution: Age of the Universe is about 14 billion years. If your age in years is
Y , then,
(Y 365.25 86400)2 = 14 109 365.25 86400
14 109
Y2 =
365.25 86400
14

103 500
28

500 22 years
Y
Thus, your age is in early 20s.

2. If f (x + y) = f (x)f (y) and f (2) = 5, then the value of f (2) is


A. 5

B. 1

Solution:

C. 0.25

D. 0.2

HB

Solution 1:
f (4) = f (2)f (2) = 25 and f (2) = f (4 2) = f (4)f (2) = 25f (2)
f (2)
5
Thus: f (2) =
=
= 0.2
25
25

Solution 2:
f (2) = f (2 + 0) = f (2)f (0) implying f (0) = 1
Thus, f (0) = f (2 2) = f (2)f (2)
1
f (0)
= = 0.2
Giving, f (2) =
f (2)
5

3. A container with uniform rectangular cross-section and weight M kg, falls from a
cargo ship into the sea and is floating with x part of its height under water (x is a
fraction less than 1). Two persons from a nearby boat, each weighing m kg board on

INAO 2012

the container resulting it to go down such that the top surface of the container levels
in terms of x.
exactly with the water level. Find the ratio M
m
A.

(1 x)
2x

B.

x
(1 x)

C.

(1 x)
x

D.

2x
(1 x)

Solution: If V is volume of the container,

CS

M = xV
& M + 2m = V
M
M + 2m =
x
2mx = M(1 x)
2x
M
=

m
(1 x)

4. An electron moving uniformly in space is neither deflected nor accelerated over a long
distance. Which of the following statements may describe the local conditions?
A.
B.
C.
D.

Both electric and magnetic fields are necessarily zero simultaneously.


The electric field is necessarily zero.
The magnetic field is necessarily zero.
Neither of them are necessarily zero.

Solution: The Lorentz force equation is given by,


~ + ~v B)
~
F~ = q(E

HB

We require F~ to be zero. The electric and magnetic fields can be so adjusted that
their net effect cancels out, and the electron continues its path.

5. Pradip once observed image of the Sun on the platform of CST station (Mumbais
main railway station). He realised that the sunrays forming the image are entering through a tiny triangular shaped window in the high ceiling. He measured the
diameter of the image to be 0.175 m. Find the height of the ceiling.
A. 12.5 m

B. 18.75 m

C. 22.5 m

D. 37.5 m

Solution: h = height of the ceiling; Dse = 1.5 1011 m ;w = diameter of image;


d = 14 108 = diameter of sun.
Dse
h

w
d

1.5 1011
14 108
h 18.75m
h .175

INAO 2012

6. The most energy efficient direction of projection of rockets from the earth surface is
A. Eastwards

B. Westwards

C. Northwards

D. Vertically upwards

Solution: The earth rotates from west to east. If the rocket is projected in a
eastward direction, the earths motion acts as a boost for the projection.
7. A car travels the first 20 km eastwards with 30 km/hr, next 20 km northwards with
40 km/hr and then 20 km westwards with 50 km/hr. The average velocity of the car
for the journey is
600
400
10
km/hr C.
km/hr D.
km/hr
A. 10 km/hr B.
9
47
47

CS

Solution: Total displacement is 20 km only.

total displacement
total time
20 20 20
totaltime =
+
+
30 40 50
47
hr
=
30
20
Avg.Velocity = 47
Avg.Velocity =

30

600
=
47

HB

8. The Sun is found to be setting exactly at 6.00 pm on a given day. If the Earths
atmosphere was only half dense as it is then the sunset would have occurred
A. Slightly later than 6.00 pm

B. Slightly earlier than 6.00 pm


C. Exactly at 6.00 pm

D. It depends on the latitude of the place.

Solution: We see the sunset a few minutes after geometric sunset as the solar
disk remains visible even after going below horizon due to refraction. If the
atmosphere is rarer, the sunrays will get refracted less. Thus, this additional
period of visibility of solar disk is reduced. Hence, we will see sunset slightly
earlier.

9. A knock-out tennis tournament begins with a total of 193 players. In each round, if
the number of players is odd, then one player gets direct bye to the next round. No

INAO 2012

player can get byes in two consecutive rounds. How many matches must be played
in the tournament before the winner is decided?
A. 386

B. 192

C. 169

D. 97

Solution: The only way a player can go out of the tournament is by losing.
Winner is a single player; thus, 193 1 = 192 players must lose. Implying 192
matches.

A. 45

B. 60

CS

10. Four metal rods all of identical dimensions and made of same material are welded
together at a single point. The configuration is such that any two rods are oriented
at 120 . The far ends of three of the rods are maintained at 90 temperature and that
of the fourth rod is maintained at 30 temperature. The temperature of the junction
point is
C. 75

D. None of these.

Solution: For equilibrium condition, heat flow in and out of junction should
be same. Let k represent some constant representing heat content, which may
depend on dimensions and material of the rods.
k(T 30) = 3k(90 T )
4T = 300
T = 75

Section B: (2 questions 5 marks each)

HB

11. Consider the following statements:

A central eclipse is the one where central point of lunar disk exactly passes over
the central point of solar disk.

An eclipse is called a total Eclipse if it is seen as total from at least some point
on the earth.
An eclipse is called a partial Eclipse if it is not seen as total / annular from any
point on the earth.

Now choose the incorrect statement/s from below:


A. All central eclipses are total.
B. All total eclipses are central.
C. All partial eclipses are non-central

D. All non-central eclipses are partial

INAO 2012

Solution: First one is incorrect as some central eclipses can be annular only.
Second one is incorrect as one can get a total eclipse even when the two disks are
slightly off-center w.r.t. to each other.
Third one is correct.
Fourth one is incorrect for the reason explained above regarding the second option.
12. Consider a pendulum with inextensible string with a lightweight magnetic bob. Underneath this arrangement iron dust is spread out. Now the bob of the pendulum
starts swinging. Which of the following might be a susequent observation(s)?
A. The maximum height reached by the bob will start increasing.
B. The tension in the string will start increasing.

CS

C. The period of oscillation of the bob will start decreasing.


D. The angular momentum of the bob remains constant.

Solution: As the oscillation starts the bob will slowly pick up iron dust and its
mass will increase. As the velocity of the bob at the hightest point is zero, by
conservation of energy, height of the highest point must decrease.
As the mass increases, the tension in the string will increase too.
Since the string is inextensible the time period will remain the same.
But angular momentum, L = I will change, as I changes.(I is moment of inertia).

Section C: Analytical Questions

HB

. (10 marks) In the night sky, the constellations depict several shapes of animate objects (species like birds, animals, etc) or man made instruments. List 20 such figures
/ instruments. The list should contain at least 4 instruments. Write both the object
and the corresponding constellation. Exact name of the constellation is not necessary.
The constellations must be from International Astronomical Unions standard list of
88 constellations (i.e. Indian / Chinese / Mayan constellations not accepted).
Solution:

Animals (easy): bull (Taurus), ram (Aries), eagle (Aquila), bear (Ursa Major
/ Ursa Minor), lion (Leo), crab (Cancer), scorpion (Scorpio), dog (Canis Major / Canis Minor), fishes (Pisces), swan (Cygnus), dolphin (Delphinus), snake
(Serpens), crow (Corvus), man (Orion / Bootes etc.)
Animals (difficult) - : fly (Musca), hare (Lepus), Wolf (Lupus), fox (Vulpecula), peacock (Pavo), dove (Columba), Emu (camelopardalis), small horse (Equuleus), swordfish (Dorado), crane (Grus), lizard (Lacerta), Chamaeleon, hornbill
(Tucana)

INAO 2012

Animals (mythical): water goat (Capricornus), centaur (Centaurus), dragon


(Draco), winged horse (Pegasus), a mythical snake (Hydra), a sea monster (Cetus), a bird (Apus), Phoenix

instruments: Telescopium, scales (Libra), Microscopium, Sextans, directional


compass (Pyxis), Table (Mensa), cross (Crux), Crown (Corona Australis / Corona
Borealis), harp (Lyra), arrow (Sagitta), shield (Scutum), easel (Pictor), air pump
(Antila), compasses (Circinus)
Each correct creature / instrument: 0.5 marks. Maximum 16 creatures allowed.
. From the graph below, find:

(a) (7 marks) The cubic polynomial describing the curve. [Note: f (7) 6= 200.]

CS

(b) (5 marks) If this curve describes orbit of a comet with the Sun at point (1,0)
and the comet crosses the x-axis exactly two months apart, find the approximate
position of the comet 1 month since first crossing of the x-axis.

HB

880
840
800
760
720
680
640
600
560
520
480
440
400
360
320
280
240
200
160
120
80
40
0
-40
-80
-120

-5

-4

-3

-2

-1

Solution: (a) From the graph, we note that f (4) = 0, f (8) = 0, f (0) = 800

10

INAO 2012

and f (1) = 840. Using these to find the cubic polynomial,

ax3 + bx2 + cx + d
a(4)3 + b(4)2 + c(4) + d = 64a + 16b 4c + d
83 a + 82 b + 8c + d = 512a + 64b + 8c + d
d
a+b+c+d
a+b+c
576a + 48b + 12c
48a + 4b = 4(12a + b)
64a + 8b + c + 100
24b 3c + 900
8b + 300

=
=
=
=
=
=
=
=
=
=
=

CS

f (x)
0
0
800
840
40
&0
c
&0
0
c

(2.5 marks)

Solving, we get,

f (x) = x3 29x2 + 68x + 800

(1)

(b) We have to divide the area enclosed by the curve and the x-axis in 2 equal
halfs as divided by the line from point (1,0), to get the position for the comet
after one month.
Non-calculus solution: A rough counting of squares shows that the correct
position would be approximately between x = 2 and x = 2.5.
Calculus based solution: If the x-coordinate of the comet after one month was
x0 ,
Z x0

Z 8
1
f (x)dx (x0 1)f (x0 )
f (x)dx = 2
(2)
2
4
4

HB

Solving, we get,

88
x40 32x30

+ 29x20 868x0 + 64
=0
2
3
3

(3)

x0 = 2 yeilds slightly positive value and x0 = 3 yeilds large negative value. Thus,
the root the the equation lies very close to 2 but is higher than 2.
Note: The approximate solution turns out to be x0 = 2.2071.
Marking Principle: By either way, student should state (with proper justification)
that the x-coordinate is close to 2 but higher than 2.

. (a) (6 marks) Assume that human civilisation has carved out all the mantle and
the central core of the earth, and started living on the inside surface of the
earths crust which is only about 7 km in thickness. As a consequence, people
are finding that their weight has altered drastically. To feel normal weight again,
one scientist suggested an idea involving tampering one of the motions of the
earth. Can you describe this idea qualitatively and quantitatively?

INAO 2012

Solution: For a person standing on the inside surface of the hollow shell,
there is no effect of gravity due to the shell itself (shell theorem).
He / she would feel the weight purely due to the centrifugal force due to
the rotation of earth.
(2 marks)
Thus, to get the normal weight back, the earth must start rotating faster.
(0.5 mark)
Shell thickness should be considered as negligible as compared to the radius
of the earth.
(0.5 mark)

CS

g = R 2
r
g
2
=
=
T
R
s
R
T = 2
g
r
6.4 106
2
10
2 800
T 5000 sec

Thus, the earths rotation period should be decreased to about 84 minutes.


(2 marks)
This calculation is valid only for the equator and apparent weight at other
latitudes would be different.
(1 mark)

HB

(b) (8 marks) An observer is sitting in a stationary spacecraft outside earths orbit


such that at the closest point, distance between the observer and the earth is 1
AU. Sketch a rough plot (exact functional form / shape not expected) showing
how the orbital angular velocity of the earth varies with time (in an year), as
seen by this observer. Mark every point of maximum or minimum clearly and
write its coordinates. Similarly, mark every point with zero angular velocity and
write its coordinates. Assume the orbit of earth around the Sun to be circular.
B

Solution:

observer

60

Earth

Sun

C
R

CS

INAO 2012

The exact expression for the function above is

sin() cos( + )
sin()
R sin()
where tan =
k + R(1 cos())
() = 0

In this, R is the orbital radius of the earth (1 AU), k is the shortest separation
between the observer and the earth orbit (also 1 AU), is the observer - sun earth angle, is the earth - observer - sun angle. The coordinates for points of
interest would be (0, 0 ), (60,0), (180, 0 /3) and (300,0), where x-axis represents
. This can be recalibrated in terms of time with 30 interval corresponding to
one month period.

HB

The sign of as well as start of the year point are matters of convention. Hence if
this graph is flipped on x-axis or is shifted along the x-axis, the answer is equally
valid.
Marking scheme: 1.5 mark for each the four coordinates. 2 marks for the overall
shape of the curve and proper markings of axes etc.

. In the 1957 science fiction novel The black cloud by Sir Fred Hoyle, an interstellar
dark cloud is discovered appraching the solar system. We will try to retrace the
steps explained in the novel to calculate speed and direction of the cloud. Two
photographs given here show a small region inside the constellation of Orion. The
photographs are taken exactly 2 months apart. The grid in the image has the size of
1 1 . By spectroscopy, it was realised that the emission line of neutral hydrogen
(Rest wavelength 21.1000 cm) emitted by this cloud was showing up at 21.0789 cm
wavelength.

CS

INAO 2012

(a) (4 marks) In how many months (after the second image) the cloud will entirely
cover the belt of Orion (seen on the left)?
(b) (1 mark) Is the cloud headed directly for the solar system? Why?
(c) (2 marks) If yes, then in how many months (after the second image) the cloud
will arrive at the earth? If no, what will be the closest separation between the
cloud and the earth? Assume the cloud to have uniform velocity throughout the
journey.
(d) (5 marks) If this spherically symmetric cloud is placed at the exact centre of
the solar system, which planets will be engulfed by the cloud?
r
cv
Note: The expression of relativistic Doppler effect is given by observed = emitted
c+v

HB

Solution: (a) The size of the cloud is about 1 cm in the first image and 2 cm
in the second image. Now, Mintaka and Alnitak are almost equidistant from the
clouds centre (about 4.6 cm) so they would be get covered at almost same time.
If the physical diameter of cloud is x and its distace from us is d,
x = d = 2(d vt2 ) = 9.2(d vt3 )
d = 2vt2
& 9.2vt3 = 7.2d + 2vt2 = 16.4vt2
16.4
t3 =
2 months
9.2
= 3.565 months

i.e. In just over six weeks from the second image (about 47 days), the belt of
Orion will be covered by the cloud.
(b) The cloud is headed directly for the solar system as the position of its centre
has not shifted from the first image to the second image.

10

INAO 2012

(c)

d = 2vt2 = vt4
t4 = 2t2
t4 = 4 months
Thus, the cloud will arrive at the solar system in exactly 2 months after the
second image is taken.
(d) Velocity of the cloud is given by,
2 
2
observed
21.1 (1 0.001
=
= 0.998
emitted
21.1
2c
300km/s
v =
1998


CS

cv
=
c+v

As per the grid, about 1.95 cm correspond to 1 separation in the sky. Thus,
0.5 in the first image.
x = d = 2vt2
2 3 105 2 30 86400

x =
0.5
11
1.5 10
180
24 8.64

360

1.728
3
x 1.8 A.U.

HB

Thus, the cloud radius would be about 0.9 A. U. Meaning, the cloud will engulf
Mercury and Venus.
. (12 marks) Let us assume a situation where an asteroid passes close to the earth.
Some times the shortest distance of such an asteroid can be even lesser than the earthmoon distance. In such cases, the asteroid will cross the lunar orbit at two points. A
particular asteroid makes a close-earth pass such that at its closest proximity to the
earth, its distance from the earth is 256000 km. For simplicity, let us assume that the
gravitational force of the earth acts on this asteriod only for a short duration of 1000
seconds, when the asteroid is exactly at the closest proximity point. Let us assume
that the asteroid follows straight line paths with uniform velocities before and after
this gravitational interaction and the velocity of the asteroid before the interaction
was 6.25 km/s. Find out how much would be the deviation measured (in km) along
the lunar orbit. Assume the Moon to be on the other side of the Earth in its orbit
during the asteroids transit, hence not having any impact on the motion. Take the
orbit of the Moon around the Earth to be circular.
Solution:

11

INAO 2012

Path if
undeviated
B

Path after deviation


r

Moon

Earth
d
(Shortest
distance)

Approach

~v = ~u + ~at
GMt
~v = uj + 2 i
d

(3 marks)

6.67 1011 6 1024 103


i
(2.56 108 )2

CS

= 6.25 103j +

6250j + 6.1i

(3marks)

As the first term on the R.H.S. is much larger as compared to the second term,
the angle of deviation of the asteroid would be very small. In the figure, E denotes
centre of the Earth, A closest proximity position of asteroid, BC is arc measured
along the lunar orbit. AB is denoted by r and both EB and EC would equal to
R. As angle of deviation is very small, Arc length BC can be approximated as
length BD. Let angle BAC be and angle AEB be . Thus, angle FBC and hence
approximately angle FBD will also be 90 . Angle BFD can be approximated
to 90 .
tan() =

HB

tan()

at
103
u
103
r
R
BF
cos(90 )
r tan()
sin()
R tan()
3.84 108 103
384km
(6marks)

sin() =
BD =
=

Note: If BC is approximated to BF, only 3 marks would be awarded. Secondly,


AB = r 6= R. That approach does not carry any points.

12

INAO 2012

HB

CS

Space for Rough Work

prepared using LATEX2

Indian National Astronomy Olympiad 2013


Question Paper

Roll Number:

Roll Number

Date: 2nd February 2013


Maximum Marks: 100

INAO 2013
Duration: Three Hours
Please Note:

Please write your roll number on top of this page in the space provided.

Before starting, please ensure that you have received a copy of the question paper
containing total 4 pages (8 sides).
There are total 10 questions. Maximum marks are indicated in front of each question.

CS

For all questions, the process involved in arriving at the solution is more important
than the answer itself. Valid assumptions / approximations are perfectly acceptable.
Please write your method clearly, explicitly stating all reasoning.
Blank spaces are provided in the question paper for the rough work. No rough work
should be done on the answer-sheet.
No computational aides like calculators, log tables, slide rule etc. are allowed.
The answer-sheet must be returned to the invigilator. You can take this
question booklet back with you.
Please be advised that tentative schedule for OCSC in astronomy is from 2nd to
19th May 2013.

Useful Physical Constants

HB

Mass of the Earth


Radius of the Earth
Mass of the Sun
Radius of the Sun
Radius of the Moon
Distance to the Moon
Mass of Jupiter
Astronomical Unit
Light year

Solar Luminosity
Gravitational Constant
Gravitational acceleration
Wiens constant

ME
RE
M
R
Rm
dm
MJ
1 A. U.
1 Ly
1 Ly
L
G
g
K

6 1024 kg
6.4 106 m
2 1030 kg
7 108 m
1.7 106 m
3.84 108 m
2 1027 kg
1.5 1011 m
9.461 1015 m
63240 A.U.
3.826 1026 W
6.67 1011 m3 /(Kg s2 )
9.8 m/s2
2.898 103 m.K

HOMI BHABHA CENTRE FOR SCIENCE EDUCATION


Tata Institute of Fundamental Research
V. N. Purav Marg, Mankhurd, Mumbai, 400 088

INAO 2013

1. (10 marks) Astrophysicist Freeman Dyson proposed around 50 years back that an
advanced civilisation would make optimal use of the energy of the parent star by
constructing a full shell around the parent star with radius equal to the orbital
radius of their planet, trapping all the radiation inside. The civilisation can live
on the surface of this shell. We would like to construct a Dyson sphere with radius
equal to the orbital radius of the Earth. Let us assume that we have access to all
the material within the solar system (except the Sun itself, which we would like to
retain as the energy source) and we have necessary technology to solidify the material
available to us. What will be rough thickness of this shell?

CS

Solution: Total mass available to us for constructing the Dyson Sphere would
primarily consist of mass of all planets and satellites. As an approximation, we
will say Jupiter and Saturn are of similar size. Saturn is known to have very low
density so its mass will not exceed half Jupiter mass. Further, radius of Uranus
and Neptune is less than half of Jupiter radius. Thus, they will be at least 10
times lighter than Jupiter. Similarly, Earth and Venus are of similar size and
Mars is about half of that. There are a handful of objects with radii about a
quarter of that of Earth or even smaller (Mercury, Moon, Ganymede, Callisto,
Titan etc.). All these are rocky bodies and we can take typical density to be
s = 5 gm/cc. Combined mass of all other combined bodies will not be greater
than mass of Mars.
From real planetary data, total mass estimate of about 1.5MJ is closer to reality.

HB

As we want shell to be solid like rock, we will assume similar density for the shell
too. So if the shell has thickness l,


MJ
M
M
2
4D ls 2MJ + 2
+ 2M + 2
+6
10
10
100
2MJ + 0.2MJ + 2M + 0.2M + 0.06M
l
4D 2 s
2.2(MJ + M )

4(1.5 1011 )2 5 103


2.2(2 1027 + 6 1024 )

4 2.25 1022 5 103


2006
10

200

3 mt.
Thus, the thickness will be approximately of the order of a few meters.

As is evident from the numbers, mass estimates are just rough indicators of total
mass. and density can be lowered to about 2.5 gm/cc. Thus, the thickness can
go up to 5 metres.
Marching Scheme:
Reasonable estimate of mass ( 1.5 3.0MJ ) and density (1 - 10 gm/cc) with

INAO 2013

reasons: 5 marks
Rest of the Calculation: 5 marks

Solution:

CONSTELLATION

Orion
Cygnus
Taurus

Virgo
Scorpio

CS

STAR
Polaris
Rigil Kentaurus

Sirius
Vega
Regulus

2. (10 marks) Following table contains names of a few constellations and names of some
stars. Fill in the blanks. For missing star names, you can name any star from that
constellation.

CONSTELLATION
Ursa Minor
Centaurus
Orion
Cygnus
Taurus
Canis Major
Lyra
Leo
Virgo
Scorpius

HB

STAR
Polaris
Rigil Kentaurus
Betelgeuse
Deneb
Aldebaran
Sirius
Vega
Regulus
Spica
Antares

3. (10 marks) An Indian festival called Kojagiri is always celebrated on a Full Moon
night in Autumn. As a part of festivities, many people keep a bowl of milk on the
ground or a terrace under the Moon light for a few minutes, before drinking it. On
this years Kojagiri night, Sheetal had placed roughly 100 ml milk in a dish of size
25cm diameter for about 10 minutes exactly when the moon was overhead. Assume
that density of milk is similar to the water, its specific heat capacity is 0.9 times that
of water and heat loss from the milk to surroundings is negligible. Assume that full
Moon reflects about 14% of the sunlight incident on its surface. What will be the
change of temperature caused by the light received from the moon?

INAO 2013

Solution: Moon receives light energy from the Sun which is partly reflected back
towards the Earth.
L
(1 mark)
4D 2
L
2
Total Solar Energy incident on Moon =
Rm
(1 mark)
2
4D
L
2
Rm
(1 mark)
Total Solar Energy reflected by Moon =
2
4D
2
L Rm

Reflected Energy received at Earth per unit area =


= Er (2 marks)
2
4D 2d2m

Solar Energy per unit area incident on Moon =

Let us assume that this entire energy is used to heat the milk without any losses.

CS

tEr A = msT (1 mark)


2
tr 2
L Rm

(1 mark)
T =

2
V s 4D 2d2m

600 0.1252 3.826 1026 (1.7 106 ) 0.14


=
(1 mark)
104 103 0.9 4.18 103 8 (1.5 1011 )2 (3.84 108 )2
1
1.4 104 (2 marks)
T
7000
Thus, temperature will rise by only about 104 Celsius.

4. (7 marks) Let A and B be two objects in the solar system orbiting each other and
A is much heavier than B. Theoretically, what is the maximum possible revolution
period of B?

HB

Solution: To have longest possible period, the orbital radius should also be
longest. If A was any planet, orbital radius of B cannot be very high as far
from the planet B will come under influence of the Sun. So for longest possible
radii, we take A as the Sun and try to place B as far as possible.
(1.5 marks)
The other limiting factor will be the influence of other stars. As B is still part of
the solar system, Sun exerts more influence on it than any other star.
(1.5
marks)

Star closest to us is the Alpha-Centauri triplet. It has two stars similar to the
Sun and Proxima Centauri, which is much smaller than the Sun. Thus, we can
approximate the system as about 2M system about 4.3 light years away. Hence,
rd
at about 13 of the distance, Alpha-Centauri system will start dominating on B.
4.3
This means that maximum possible radius is about
= 1.4 light years. Thus,
3
longest period will be,
(1.5 marks)
We can utilise the fact that orbital radius of the Earth is 1 A.U. and orbital

INAO 2013

period is 1 year.

T12
=
T22
T1 =

4 2 3
r (0.5 marks)
GM
R13
R23
s
R13
T2
R23
s
(1.4 63240)3
1
13
q
(90000)3

T2 =

CS

= 3003
= 2.7 107 years (2 marks)

Thus, approximate period would be 27 million years.

It is acceptable if Alpha-Centauri system is assumed to have mass of just a M .


5. (8 marks) Prove that 2013 20124 + 1 is a composite number.
Solution:

2013 20124 + 1
(2012 + 1) 20124 + 1
(a + 1) a4 + 1 where a = 2012
a5 + a4 + 1 (2 marks)
a5 + a4 + a3 a3 + 1
a3 (a2 + a + 1) (a3 1)
a3 (a2 + a + 1) (a 1)(a2 + a + 1)
(a2 + a + 1)(a3 a + 1)
(20122 + 2013)(20123 2011) (6 marks)

HB

X =
=
=
=
=
=
=
=
=

6. (10 marks) Ayush was doing an experiment with a lens in his Physics lab. He kept
a reference object at several positions and noted corresponding image sizes and positions. The same are shown in the diagram below. Unfortunately, he forgot to label
the object and image positions and to mention details about the lens, because he
thought them to be too trivial. Help his science teacher to make sense of his drawing
by finding type of lens (convex / concave), its position and its focal length. In the
diagram, O stands for Object and I for image. All lengths and heights are to scale.

INAO 2013

I
I
I

I
O

CS

Solution: The type of lens is convex lens. In the picture, Object-Image pairs
have been given same subscript. O5 is placed at exact focus so it has no corresponding image. Focal length of lens is about 2.25 cm (From the figure, 4f = 9.1
cm).

I4

I
3

I2

I
1

O4

HB

O2

Marking scheme: Lens type - 1 mark, Reasoning - 2 marks, position - 3 marks,


focal length - 3 marks, cross checking - 1 mark

7. (5 marks) In a competition for the most useless inventions, Sharad presented a novel
technique which makes it possible to fold an ordinary paper 50 times on itself, without
any gaps between layers of folds. Calculate thickness of the folded paper after it has
been folded 50 times. Note that the paper is folded further and further without
opening the previous folds i.e. it is folded in half and then the folded paper is folded
on the middle to reduce the area to a quarter and so on.
Solution: A 100 page notebook is roughly 1 cm thick.
(1 mark)
2
4
4
Thus, thickness of a single paper is 10 cm or 10 mt (2 10 2 105 mt
accepted).
(1 mark)

INAO 2013

If you fold the paper 50 times, its thickness will become,

l = 250 104 (2 marks)


5
= 210 104 = (1024)5 104
1015 104
1011 mt. (1 mark)

(1)
(2)
(3)
(4)

Thus, the thickness will be of the order of the Sun-Earth Distance.

CS

8. (10 marks) Alankar was jogging downhill and he noted distance traveled by him at
various intervals. Plot appropriate graph to find his acceleration and initial velocity.
In the table, time (t) is in minutes and distance (d) is in meters. Both the quantities
are measured from the start of the journey.
t d
2 21
4 60

t d
6 117
8 192

t
d
10 285
12 396

t
14
16

d
525
672

t
d
18 837
20 1020

Solution: d = ut + 21 at2
Thus, if one plots a graph of distance versus time, it will be a parabolic curve.
d
a
To linearise the graph, we change the equation as = u + t
t
2
a
d
Now if we plot versus t, it will give a linear graph with as slope and u as
t
2
y-intercept.
t d/t
6 19.5
8 24

t d/t
10 28.5
12 33

HB

t d/t
2 10.5
4 15

t d/t
14 37.5
16 42

t d/t
18 46.5
20 51

Thus, u = 6 m/min and a = 4.5 m/min2 .


Marking scheme: Linearisation of equation: 2 marks, graph plotting skills: 4
marks, slope: 2 marks, final answers: 2 marks. Solutions with non-linear graphs:
maximum 5 marks.

INAO 2013

9. (10 marks) Sandesh fabricated a magic sphere of radius R, which is hollow on the
inside and has perfectly reflecting inner surface. This sphere had a small hole in it.
Sandesh sent a ray of light radially through this hole. It hit a plane mirror, kept at
an angle of 45 with the incident ray, at some point beyond the centre of the sphere,
but before reaching the opposite end. After undergoing one more reflection at the
inner surface of the sphere, the ray came out from the hole. Find the distance from
the centre of the sphere to the point where it struck the plane mirror.

CS

Solution: In the figure below, D is the point of second reflection. OD will be


normal to the surface at D.

HB

OBD = 90 (1 mark)
BDO = ADO = DAO = x AO = OD (6 marks)
AOD = 180 (ADO + DAO)
= 180 2x
but AOD = OBD + BDO
180 2x = 90 + x
x = 30 (2 marks)
R
l(OB) = l(OD) sin 30 =
(1 mark)
2

10. (20 marks) Answer following questions in 3 to 4 lines each.


(a) The core of earth is like a perfectly conducting fluid sphere. If this core suddenly
shrinks to half of its present radius, then what will happen to its magnetic field?
Why?
7

INAO 2013

(b) A satellite is revolving around the Earth in a polar orbit with 90 minute period.
On 23rd September, Prasad saw it exactly overhead at the time of sunset. Within
next twenty four hours how many sunrises will be seen by the satellite? Why?
(c) Anand has taken a high zoom photograph of a sun-spot. The photograph does
not include any part of the disk outside sun-spot. What will be the predominant
colour seen in the photograph? Why?

(d) What will happen to the Moon if the Earth vanishes suddenly? Why?
(e) On a full Moon day, with respect to an observer on the Earth, does the Moon
move faster (angle covered per hour) during the day time or the during night
time? Why?

CS

1
Solution: (a) Magnetic flux must be conserved. Thus, B 2 . As the radius
R
shrinks to half, the magnetic field will increase four-fold.
(b) As the satellite makes angle of 90 with the Sun, its orbit is exactly along the
line separating day and night regions on the Earths surface. Hence, its one side
will continuously face the Sun and hence, there will be NO sunrises seen by the
satellite.
(c) The sunspots have temperature of around 4500K. By Wiens law, this corresponds to roughly red colour. Thus, the sunspots will appear red.
(d) Moon will retain all its velocity at the instance of disappearance of the Earth.
Thus, it will have significant non-radial velocity around the Sun and it will continue to orbit the Sun in near identical path.

HB

(e) We note that Moon revolves in the same direction as the rotation of the Earth
i.e. from West to East. On the full Moon day, Moon is on the night side of the
Earth. Thus, with respect to an observer on the Earth, the tangential velocities of
the Moon and the Earth during the night will be in the same direction and hence
relative velocity is slower. During the day tangential velocities are in opposite
directions hence the relative velocity is faster. Thus, Moon will be moving faster
during the day.

Space for Rough Work

INAO 2013

Notes for Junior Group


Question 4: Take body A to be the Sun. Keplers third law states that the square
of orbital period will be proportional to the cube of orbital radius.
Question 5: a3 1 = (a 1)(a2 + a + 1).

Question 8: Modify appropriate equation of motion such that all points will lie
along a straight line.
Question 10 (a): If the conducting sphere suddenly changes the size, the field lines
will stay trapped on the surface.

CS

Question 10 (c): At any given temperature, the peak of thermal radiation will be
Wien s constant
at wavelength max =
.
T

Notes for Junior Group

Question 4: Take body A to be the Sun. Keplers third law states that the square
of orbital period will be proportional to the cube of orbital radius.
Question 5: a3 1 = (a 1)(a2 + a + 1).

Question 8: Modify appropriate equation of motion such that all points will lie
along a straight line.
Question 10 (a): If the conducting sphere suddenly changes the size, the field lines
will stay trapped on the surface.

HB

Question 10 (c): At any given temperature, the peak of thermal radiation will be
Wien s constant
at wavelength max =
.
T

Notes for Junior Group

Question 4: Take body A to be the Sun. Keplers third law states that the square
of orbital period will be proportional to the cube of orbital radius.

Question 5: a3 1 = (a 1)(a2 + a + 1).


Question 8: Modify appropriate equation of motion such that all points will lie
along a straight line.
Question 10 (a): If the conducting sphere suddenly changes the size, the field lines
will stay trapped on the surface.
Question 10 (c): At any given temperature, the peak of thermal radiation will be
Wien s constant
at wavelength max =
.
T

prepared using LATEX2

Indian National Astronomy Olympiad 2014


Question Paper

Roll Number:

Roll Number

Date: 1st February 2014


Maximum Marks: 100

INAO 2014
Duration: Three Hours
Please Note:

Please write your roll number on top of this page in the space provided.

Before starting, please ensure that you have received a copy of the question paper
containing total 3 pages (6 sides).
There are total 8 questions. Maximum marks are indicated in front of each question.

CS

For all questions, the process involved in arriving at the solution is more important
than the answer itself. Valid assumptions / approximations are perfectly acceptable.
Please write your method clearly, explicitly stating all reasoning.
Blank spaces are provided in the question paper for the rough work. No rough work
should be done on the answer-sheet.
No computational aides like calculators, log tables, slide rule etc. are allowed.
The answer-sheet must be returned to the invigilator. You can take this
question booklet back with you.
Please be advised that tentative dates for the next stage are as follows:
Orientation Camp (Junior): 28th April to 6th May 2013.
Orientation Camp (Senior): 1st May to 6th May 2013.

Selection Camp (Jr. + Sr.): 27th May to 6th June 2013.

HB

Participation in both parts (Orientation and Selection) is mandatory for all


participants.

Useful Physical Constants

Mass of the Earth


Radius of the Earth
Mass of Jupiter
Astronomical Unit
Gravitational acceleration
Avogadro constant
Solar Constant

ME
RE
MJ
1 A. U.
g
Na
S

6 1024 kg
6.4 106 m
2 1027 kg
1.5 1011 m
9.8 m/s2
6.023 1023 mol1
1400 W/m2

HOMI BHABHA CENTRE FOR SCIENCE EDUCATION


Tata Institute of Fundamental Research
V. N. Purav Marg, Mankhurd, Mumbai, 400 088

INAO 2014

1. (12 marks) Read the following passage and point out scientific inaccuracies. Give
very brief argument for each mistake you point out.

Arrival of a spacecraft on the surface of the Moon is called a Moon landing.


This includes both manned and unmanned (robotic) missions. In order to get
to the Moon, a spacecraft must first cross the point, beyond which the Earths
gravitational force is zero. The only practical way of accomplishing this currently
is with a rocket. At every instant, the rocket has to produce thrust to propel
itself to a velocity equal to critical velocity at that height. Jet engines can be
used to propel the spacecraft to the Moon, but rockets provide greater advantage
in terms of power required for given mass of the spacecraft.

CS

Unlike the Earth, the Moon does not have thick atmosphere to absorb most of
the solar radiation and the magnetic field of the moon is too weak to deflect
the UV rays. Thus, astronauts traveling to the Moon are exposed to harmful
electromagnetic radiation. Spacesuits for astronauts are specially designed by
keeping this in mind.
On arrival near the Moon, the spacecraft is captured by the lunar gravity in an
orbit around the Moon. As the Moon is much smaller than the Earth, typically
these orbits are low altitude orbits (close to the lunar surface) as compared to the
polar satellites around the Earth. The spacecraft can stay in this orbit forever,
however, if one needs it to land on the Moon, it has to fire its engines to change
course.

HB

Landing at the Moon can be of two types. A hard landing is equivalent to crash
landing on the Moon. A soft landing is a controlled landing in which priority
is to maintain all equipment and astronauts (if any) inside the spacecraft safe.
Needless to say, all Apollo missions had a soft landing on the Moon. This was
achieved by firing reversing rockets very close the lunar surface to slow down the
spacecraft. Charring of the lunar soil, due to burning of these rockets, is one of
the permanent impressions left by humankind on the Moon.

Solution:

Earths gravitational force goes to zero only at infinity. Close to the Moon,
the force is small but not zero.
Velocity needed is bigger than the critical velocity.
For jet engines, the oxygen is taken from the air. It is not stored on-board.
So they cannot be used to take spacecraft to the Moon.
UV rays are electromagnetic waves and are not affected by the magnetic
field.
Low altitude orbits are not stable, as imperfections on the lunar surface
result in change in gravitational force from point to point.

INAO 2014

The lunar soil wont get charred due to firing of rockets as there is no oxygen
on the lunar surface.
One mark for each correct indentification of the mistake. One mark each
for corresponding reasoning.

If a blatantly wrong argument is resulting in incorrect identification of an


inaccuracy, one mark will be subtracted.
No negative marking for subtle mistakes in reasoning.

CS

2. (5 marks) A comet is in elliptical orbit around the Sun with period T and semimajor axis a. A second comet around the Sun has same period T , but a different
orbit with same eccentricity e. The two orbits are not necessarily in the same plane.
Further, we also know that when the first comet is closest to the Sun in its orbit (at
perihelion), the other comet is farthest from the Sun in its orbit (at aphelion). Given
this configuration, what is the minimum possible distance between the two comets,
when one of the comets is at its perihelion? Justify your answer by brief arguments
/ sketches.
Solution: By visualising the configuration correctly, it will be realised that to
achieve minimum possible separation, the two orbits should lie in the same plane
with their semi-major axes along the same line, such that sun - perihelion-1 aphelion-2 are collinear in that sequence.
(3 marks)
Thus, the minimum separation will be

HB

d = daphelion,1 dperihelion,2
= a(1 + e) a(1 e)
= 2ae (2 marks)

3. On a flight between the cities of Oslo (61 N, 8 E) and Helsinki (60.2 N, 25 E),
Mayank saw the Sun just at the horizon on the west. After 20 minutes, when he
checked again, he saw the Sun was now 2 above the horizon.
(a) (2 marks) The flight was traveling from which city to which city?

(b) (8 marks) What was the speed of the aircraft with respect to the ground (in
Km/hr)?
(c) (4 marks) How long will this aircraft take to traverse the distance between these
two cities? Assume uniform speed during the entire flight.

INAO 2014

Solution: The Sun rising from the west can be observed only if we are traveling in
the direction opposite to the Earths rotation. Thus, the flight was from Helsinki
to Oslo.
(2 marks)

In 20 minutes, the Earth would rotate by 5 eastwards. Mayank saw the Sun
rising by 2 in that time. This means, the plane moved (5 + 2 ) westwards in
20 minutes.
(2 marks)
We can approximate latitudes of both places to = 60 .

(2 marks)

CS

= 5 + 2 = 7

rad s1
=
20 60 180
1200 180
v = r = R cos
7
= 6.4 106
cos 60
1200 180
7
1
28
22
= 64

1.2 1.8 2
0.27
7
8800 18
326 m/s

27
5
1170 km/hr (4 marks)

Difference between longitudes of the two cities is 16 . Thus,


16 20
7
46min

t=

The flight will take about 46 minutes.

(4 marks)

HB

Note: The solution above is a simplification as is not estimated correctly. When


Maynak will see the Sun 2 above horizon, it would actually mean that at 60
latitude, the Sun has rolled back by 4 along its apparent path.
Thus, solution using = 9 is more correct. However, students are not expected
to know this at INO level and hence solution given above is accepted for full
credit.

4. The skymap below corresponds to sky above Nagpur (21 N, 79 E at 09:00 am on 1st
February 2014. If you are not used to using sky maps, it is important to note that
sky map is usually seen lying down on the ground (feet to the South), facing the sky
with map in your hand. Thus, East is on the left of the map and West is on the
right. Answer the following questions:
(a) (1 mark) Mark Polaris with letter P.

(b) (2 marks) Circumpolar stars for a given place are the stars, which will never go
below the horizon. Draw boundary of this region and mark it by the letter C.
(c) (2 marks) The celestial equator is just a projection of the Earths equator in
the sky. It will be locus of points which are equidistant from the north and the
3

INAO 2014

south pole. Draw the equator on the map approximately and mark it with Q.
(d) (2 marks) The ecliptic is the imaginary yearly path of the Sun in the sky. Mark
this approximately on the map and mark it with E.
(e) (2 marks) Mark approximate position of the Sun on the map as S.

(f) (2 marks) Yesterday was a new moon day. Mark the current position of the
moon on the map as M.
(g) (2 marks) Which star was very close to the Zenith at 06:00 am today? Mark it
on the map as N.

HB

CS

(h) (2 marks) Draw a line across sky showing horizon line as at 07:00 am today as
H.

Solution: Notes:

Pole star should be exact.

Circumpolar region should be a circle around P with radius equal to


Polaris-Northern horizon distance.
Equator should be a smooth curve passing through East and West points
and below zenith (centre) in the middle.
Ecliptic should be a smooth curve roughly passing through the zodiacal
signs (i.e. mostly below equator) and cutting equator in Virgo.

INAO 2014

The Sun should be in Capricornus.


The Moon should be in Aquarius.
Any star reasonably close to Archturus is accepted as answer for N.

CS

Horizon at 7:00am should not cut current northern horizon, roughly pass
close to the Sun and cut bottom edge on the west of south.

HB

5. (12 marks) Gliese-876 is a star in the constellation of Aquarius which has four confirmed planets going around it. We know following facts about this system:
the four planets have names b, c, d and e.

the four planets have masses (in Earth masses) of 7M , 15M , 225M and
720M . (not necessarily in the same order)

the orbital radii of the four planets (in A.U.) are 1.93, 30, 61, 124. (not necessarily in the same order)

the orbital eccentricities of the planets are 0.03, 0.06, 0.21, 0.26. (not necessarily
in the same order)
planet e is farthest from the star.

the heaviest planet has the lowest eccentricity.

the lightest planet is also the closest to the star.

the two giant planets are neither the closest nor the farthest from the star.

the planet with a mass of 7M has eccentricity of 0.21 and is closer to the star
than planet c.
5

INAO 2014

eccentricity of planet d is more than planet e, but less than c.


planet c is closer to the star as compared to planet b.

Use the information above to correctly identify mass, orbital radius and orbital eccentricity of each of the planet. For your help, you may use the grid given in the
answer sheet. The grid has six sub-grids of 4 4 size. You can tick mark in the boxes
corresponding to matching pairs and cross mark the boxes where there is no match.
Note that this grid is only for your help. It is your choice if you wish to use this
grid or some other method. Final answer must be written in the second table
below the grid. Only the values written in the table will be considered for
giving marks.

CS

Solution:

mass
720

eccentricity
0.03

orbital radius
61

225

0.26

30

0.21

1.93

15

0.06

124

One mark per value.

HB

6. (a) (9 marks) A thin convex lens, of focal length of f , is placed in y-z plane. The
principal axis of the lens is along the x-axis. A luminous square sheet is placed
in the x-y plane with its sides parallel to x and y axes. The centre of the sheet is
exactly at a distance of 2f from the centre of the lens and length of each side of
the sheet is f . The sheet is slightly rotated about the y-axis to make the entire
sheet visible from the other side of lens. Sketch the shape of the image of the
sheet and mark all relevant lengths in terms of f .
(b) (6 marks) A pin P is placed in front of a concave mirror such that its mid-point
lies on the principal axis at a distance d from the pole of the mirror. An observer
on the principal axis at a distance D, where D d, finds the pin and its real
image to overlap. When the observer moves slightly to the left of the principal
axis, the image is viewed to the right of the pin. From this observation, what
can be concluded about value of d? Justify your answer.

INAO 2014

Solution: (a) The image will look as follows:

CS

Correct drawing of object: 1 mark

Correct extremities of image on the axis: 3 marks


Top and bottom edges curved: 2 marks

left and right edges curved on correct side: 3 marks

(b) As you move you observing position laterally, the closer objects appear to
move faster than the farther objects.
Thus, when you move your eye to the left, the one closer out of the object and
the image will appear to shift to the right of other.
(3 marks)
It means that image distance is more than object distance.
(1 mark)
Hence, the object is between f and 2f .
(2 marks)

HB

7. (8 marks) Chandrayaan-1, the lunar mission launched by India in 2008, had all gold
wiring. A particular instrument on board the mission, was operating at the same
temperature as the space surrounding the Chandrayaan. Instrument specifications
demanded that no wire in the instrument should offer a resistance more than 7 m.
All the instruments on Chandrayaan were fabricated in ISRO labs at room temperature. If the radius of all wires in the instrument was 0.1mm (as measured in ISRO
lab), what should be the maximum allowable length (as measured in ISRO lab) of
any wire? Physical properties of gold:
Resistivity (at room temperature), = 2.214 108 m

Temperature coefficient of resistivity, = 0.0032 K1

Linear expansion coefficient, = 1.5 105 K1

Density, = 2 104 kg/m3

Molar heat capacity, Cp = 25 J mol1 K1

Molecular Weight, W 197 g

INAO 2014

Solution: Mass of the wire is,


M = r12 l1 = r22 l2
r12 l1 = r22 l2

Resistance of wire is given by,


l
l
= 2
A
r
2 = 1 (1 T )
l2 = l1 (1 T )
2 l2
R2 =
(2 marks)
r22
1 (1 T )l2
=
l r2
1l21

CS

R=

1 (1 T )l22
l1 r12
1 (1 T )(l1 (1 T ))2
=
l1 r12
1 (1 T 2T + 2 T 2 ) l1
=
r12
=

(2 marks)

Typical room temperature can be assumed to be 30 Celcius. Temperature in


space is typically 3 Kelvin. Thus, temperature difference will be about 300 Kelvin.
(1 mark)
Now, as 2 term is too small. Further, as is 100 times smaller than beta, it can
be ignored too.
(1 mark)

HB

1 (1 T )l1
r12
r12 R2
l1
1 (1 T )
(104 )2 7 103
22

8
2.214 10 (1 0.0032 300)
7
2
1
10
=

(1 0.96)
4
= 25 cm (2 marks)
R2

Thus, the wire can be at max 25 cm long.

8. (18 marks) We are in the year 2020 and the spacecraft Voyager-1 is at a distance of
around 200 A. U. from the Sun. It is still continuously sending information about its
speed with respect to inertial frame of the surrounding interstellar medium every 10
days. Following are the readings reported by the Voyager over course of 100 days.
8

INAO 2014

Time (days)
Velocity (m/s)
Time (days)
Velocity (m/s)

0
1000
60
1394.4

10
1061.5
70
1467.4

20
1124.4
80
1543.1

30
1189.0
90
1621.8

40
1255.4
100
1703.8

50
1323.8

Swapnil claimed that this indicates existence of an unknown body directly ahead in
the path of Voyager. Those of you who agree with him, find mass and distance of
this unknown body from Voyager. Those who disagree with him should give alternate
explanation for the data observed and justify their arguments.
Note:

For purpose of any calculations, you may ignore the influence of the Sun on
Voyager. At the end of calculations, you should give reason why this is a valid
assumption. Two points are reserved for this justification.

CS

As an approximation, assume the acceleration of the Voyager to be constant


during each 10 day period between the readings.
You may note that 8.52 72.

For calculations, you may assume one day approximately contains 90000 seconds.

Solution: We note that Voyagers velocity is changing. As we are ignoring influence of the Sun, this can only be because of presence of some unknown object
close by. By noting difference between first two readings and last two readings,
we realise that the change is not the same, i.e. acceleration is changing. We try
to estimate average acceleration and velocities for each period by,
vu
t
1
s
= = u + at
t
2

a=

vav

HB

In the table below, time is in days, velocities in m/sec and acceleration in


m/s2 . Average velocity of the Voyager is given by vav in m/s and last column is
cumulative sum of vav .
(8 marks)
t
0
10
20
30
40
50
60
70
80
90
100

v
1000
1061.5
1124.4
1189.0
1255.4
1323.8
1394.4
1467.4
1543.1
1621.8
1703.8

vu

61.5
62.9
64.6
66.4
68.4
70.6
73.0
75.7
78.7
82.0

68.3
69.9
71.8
73.8
76.0
78.4
81.1
84.1
87.4
91.2

vav

1003
1065
1128
1192
1259
1327
1398

vav

1003
2068
3195
4388
5647
6974
8372

INAO 2014

Now note that,

s = vav t
X
s=
vav 10 90000

GM
r2
GM
P
=
(r0 s)2
X
X
a1 (r0 (
s) )2 = a2 (r0 (
s) )2
1
X
X2

a1 (r0 (
s) ) = a2 (r0 (
s) )
1
2

71.8(r0 3195 900000) = 81.1(r0 8372 900000)


r0
r0
8.5(
8.4)
8 3.2) 9(
9 10
9 108
r0 9 108 (8.4 18 3.2 17)
9 108 96.8
8.7 1010 m
0.6 A U (5 marks)

CS

a=

HB

Now we use this distance to find mass of the object.


P
a(r0 s)2
M =
G
68.3 106 (8.7 1010 1003 9 105 )2 3

20 1011
68.3 (8.6)2 1024 3

2
68.3 74.0 1.5 1024
5 103 1.5 1024
7.5 1027 kg
4MJ (3 marks)
Now the object is about 300 times lighter than the Sun and about 400 times
closer to the Voyager as compared to the Sun.
Fobj
Mobj
4002
r2
= 2 Sun =
FSun
robj
MSun
300

Thus, gravitational force by the Sun on the Voyager will be about 500 times
smaller and hence can be safely ignored.
(2 marks)

prepared using LATEX2

S-ar putea să vă placă și